General Orthopedic 4 SA Exam
Quiz-summary
0 of 55 questions completed
Questions:
- 1
- 2
- 3
- 4
- 5
- 6
- 7
- 8
- 9
- 10
- 11
- 12
- 13
- 14
- 15
- 16
- 17
- 18
- 19
- 20
- 21
- 22
- 23
- 24
- 25
- 26
- 27
- 28
- 29
- 30
- 31
- 32
- 33
- 34
- 35
- 36
- 37
- 38
- 39
- 40
- 41
- 42
- 43
- 44
- 45
- 46
- 47
- 48
- 49
- 50
- 51
- 52
- 53
- 54
- 55
Information
Approved for 5 Category 1 SA Credits
Learning Objectives: At the conclusion of the General Orthopedic 4 Self-Assessment exam physician assistants should be familiar with diagnosing Pathological Fracture Through An Enchondroma, Plantar Fibromatosis, Anterior Tibialis Rupture, Epidural Lipomatosis, Knuckle Pads, Maisonneuve Fractures, Boutonniere Deformity, Transient Osteoporosis, Ruling Out A Seymour Fracture, Cervical Disk Herniation, Cervical Facet DJD, Radiofrequency Ablation For Lumbar Facet Syndrome, Multi-ligament Knee Injury, Tendon Ruptures In Rheumatoid Arthritis, Failed Rotator Cuff Repair In Young Adults, Extensor Pollicis Longus Rupture, Morel-Lavallee Lesion, Segond Fracture, Dupuytren’s Disease, Olecranon Fracture, 5th Metatarsal Apophysitis, ASIS Avulsion Fracture, Medial Condyle Fracture, Raynaud’s Disease, Dog Bite To Finger, Occult Scaphoid Fracture, Hip Arthritis And Injections, Large Swelling From Finger Arthritis, Lis Franc Fracture, Acute Carpel Tunnel After Distal Radius, Pec Tear, Traumatic Bone Marrow Edema, Irreducible Toe Dislocation, Periprosthetic Hip Fracture, Pathological Fracture, Bipartite Patella, Os Acromiale, Bimalleolar Ankle Fracture With Syndesmotic Injury, Sesamoid Fracture, Knee Fabella, Hamer Toes, Tarsal Navicular Fracture, Incarcerated Medial Epicondyle Fracture, Distal Radius Epiphyseal Fracture, Hip Arthritis, Hip Labral Tear, Stiff Digit, Superficial Thrombophlebitis, Nail Gun Injury, Distal Tibia Fracture, DRUJ Instability, Proximal Phalanx Fracture, Bennett Fracture, Osteochondral Lesion Of The Talus, and Slipped SCFE.
You have already completed the quiz before. Hence you can not start it again.
Quiz is loading...
You must sign in or sign up to start the quiz.
You have to finish following quiz, to start this quiz:
Results
0 of 55 questions answered correctly
Your time:
Time has elapsed
You have reached 0 of 0 points, (0)
Categories
- Not categorized 0%
-
You are required to complete the post-exam evaluation to receive CME credits. Please complete the post-exam evaluation here. Your CME certificate will be emailed to you shortly. If you don’t receive your certificate today, and the certificate email from CME4PAC is not located in your spam folder, please email, Dagan Cloutier, at dcloutier@cme4pac.com.
- 1
- 2
- 3
- 4
- 5
- 6
- 7
- 8
- 9
- 10
- 11
- 12
- 13
- 14
- 15
- 16
- 17
- 18
- 19
- 20
- 21
- 22
- 23
- 24
- 25
- 26
- 27
- 28
- 29
- 30
- 31
- 32
- 33
- 34
- 35
- 36
- 37
- 38
- 39
- 40
- 41
- 42
- 43
- 44
- 45
- 46
- 47
- 48
- 49
- 50
- 51
- 52
- 53
- 54
- 55
- Answered
- Review
-
Question 1 of 55
1. Question
A 66 year old male presents to your office with left index finger pain after getting the finger caught in a dog leash 5 days ago. He had immediate pain when the leash twisted the finger and bruising has appeared over the last few days. AP and lateral xrays (figures 1 and 2) show a displaced intra-articular pathological fracture through the middle phalanx of the left index finger. On physical exam the index finger is swollen and ecchymotic but no deformity is present. He remembers someone telling him he had a cystic tumor in his finger 25 years ago when he had a left hand xray. The fracture mostly likely involves which type of tumor?
Correct
The patients history of a bone tumor found in the finger 25 years ago is an important clue that the tumor is benign. Radiographic appearance of a well-defined (clear cortical boarder) is further evidence the tumor is not aggressive or malignant appearing. The most common tumor of the hand is an enchondroma. Enchondromas are benign slow growing cartilaginous tumors with well defined boarders and typical intralesional calcifications. Characteristics of a chondrosarcoma include cortical destruction, endosteal erosions, and soft tissue invasion. Aneurysmal bone cysts have a similar appearance to enchondromas but are bubbly appearing and don’t have intralesional calcifications. Pathological fractures through enchondromas can be treated with 1-2 months of splinting alone or curettage with bone grafting. 1,2
Answer D.
References
- Lubahn, John D. MD; Bachoura, Abdo MD. Enchondroma of the Hand: Evaluation and Management. Journal of the American Academy of Orthopaedic Surgeons 24(9):p 625-633, September 2016. | DOI: 10.5435/JAAOS-D-15-00452
- Wells ME, Eckhoff MD, Kafchinski LA, Polfer EM, Potter BK. Conventional cartilaginous tumors: Evaluation and treatment. JBJS reviews. 2021 May 1;9(5):e20.
Incorrect
The patients history of a bone tumor found in the finger 25 years ago is an important clue that the tumor is benign. Radiographic appearance of a well-defined (clear cortical boarder) is further evidence the tumor is not aggressive or malignant appearing. The most common tumor of the hand is an enchondroma. Enchondromas are benign slow growing cartilaginous tumors with well defined boarders and typical intralesional calcifications. Characteristics of a chondrosarcoma include cortical destruction, endosteal erosions, and soft tissue invasion. Aneurysmal bone cysts have a similar appearance to enchondromas but are bubbly appearing and don’t have intralesional calcifications. Pathological fractures through enchondromas can be treated with 1-2 months of splinting alone or curettage with bone grafting. 1,2
Answer D.
References
- Lubahn, John D. MD; Bachoura, Abdo MD. Enchondroma of the Hand: Evaluation and Management. Journal of the American Academy of Orthopaedic Surgeons 24(9):p 625-633, September 2016. | DOI: 10.5435/JAAOS-D-15-00452
- Wells ME, Eckhoff MD, Kafchinski LA, Polfer EM, Potter BK. Conventional cartilaginous tumors: Evaluation and treatment. JBJS reviews. 2021 May 1;9(5):e20.
-
Question 2 of 55
2. Question
A 17 year old male presents to your office with a slow growing “pea” size mass on the bottom of his left foot for the past year. He denies injury or known cause. The mass is starting to feel uncomfortable in his shoe. On physical exam there is a pea sized firm immobile mass deep to the subcutaneous tissue of the medial arch (figure 1). Coronal MRI (figure 2) shows thickening of the plantar aponeurosis measuring 1.2 by 1 cm consistent with plantar fibromatosis. What is the best treatment option?
Correct
Plantar fibromatosis is the most common benign neoplasm in the foot and is characterized by abnormal fibrous tissue proliferation and nodule formation. The condition is uncommon and typically occurs on the medial plantar aspect of the foot in older adults as a small non-painful nodule. MRI is the study of choice to confirm the diagnosis. Observation is the best treatment of choice when patients present with a small nodule with minimal symptoms. Other conservative treatments may include orthotics and local injections. Surgical excision and radiation therapy is reserved for a large painful mass that fails conservative treatment. Surgical excision alone has a high recurrence rate of 60% or greater so post excision radiation therapy can reduce this risk. A complete fasciectomy has a lower recurrence rate and is preferred compared to a local excision. 1,2
Answer A.
References
- Bos, Gary D. MD; Esther, Robert J. MD, MSc; Woll, Scott T. MD. Foot Tumors: Diagnosis and Treatment. Journal of the American Academy of Orthopaedic Surgeons 10(4):p 259-270, July 2002.
- Young JR, Sternbach S, Willinger M, Hutchinson ID, Rosenbaum AJ. The etiology, evaluation, and management of plantar fibromatosis. Orthopedic Research and Reviews. 2018 Dec 17:1-7.
Incorrect
Plantar fibromatosis is the most common benign neoplasm in the foot and is characterized by abnormal fibrous tissue proliferation and nodule formation. The condition is uncommon and typically occurs on the medial plantar aspect of the foot in older adults as a small non-painful nodule. MRI is the study of choice to confirm the diagnosis. Observation is the best treatment of choice when patients present with a small nodule with minimal symptoms. Other conservative treatments may include orthotics and local injections. Surgical excision and radiation therapy is reserved for a large painful mass that fails conservative treatment. Surgical excision alone has a high recurrence rate of 60% or greater so post excision radiation therapy can reduce this risk. A complete fasciectomy has a lower recurrence rate and is preferred compared to a local excision. 1,2
Answer A.
References
- Bos, Gary D. MD; Esther, Robert J. MD, MSc; Woll, Scott T. MD. Foot Tumors: Diagnosis and Treatment. Journal of the American Academy of Orthopaedic Surgeons 10(4):p 259-270, July 2002.
- Young JR, Sternbach S, Willinger M, Hutchinson ID, Rosenbaum AJ. The etiology, evaluation, and management of plantar fibromatosis. Orthopedic Research and Reviews. 2018 Dec 17:1-7.
-
Question 3 of 55
3. Question
A 64 year old male presents to your office 10 weeks out from a right ankle injury with continued pain and weakness. He was walking down a set of stairs and there was a crevice in the rock step and his foot went into the crevice and he felt a pop and tearing in his medial foot and ankle area. He was able to walk after the injury and didn’t seek immediate treatment as he felt it was a simple ankle sprain and would get better. Last week he had an MRI of the ankle which showed an anterior tibialis tendon rupture with the tendon stump at the level of the ankle joint. Physical exam of the right lower extremity reveals obvious palpable anterior tibialis tendon that is lodged at the ankle joint anteriorly. He as loss of normal contour of the anterior tibialis tendon compared to the contralateral ankle (figure 1). He has dorsiflexion strength rated as 4/5 at the ankle joint using the extensor hallucis longus and the extensor digitorum tendons, but no obvious palpable anterior tibialis tendon motion is noted. What is the best treatment option for this patient?
Correct
An anterior tibialis tendon rupture is an uncommon injury and is often missed initially due to the continued function of the ankle after the injury. The anterior tibialis tendon supplies 80% of ankle dorsiflexion strength so a tear will cause significant weakness. The ankle is able to dorsiflex after an anterior tibialis tendon rupture due to intact adjacent tendons including the extensor hallucis longus (EHL) and extensor digitorum longus (EDL) tendons. Symptoms of a rupture include a toe dragging gait due to dorsiflexion weakness, a mass on the anterior ankle (retracted tendon stump), and loss of anterior tibial tendon contour. Surgical treatment is recommended in most cases unless contraindicated (low demand patients and medical comorbidities). An ankle foot orthosis is a good option for patients who refuse or aren’t candidates for surgery. A primary repair can usually be performed within the first 6 weeks of injury, however a later presentation usually requires a tendon graft reconstruction or local tendon transfer. Tendon transfers are generally reserved for patients who can’t have a primary repair or graft reconstruction. Surgical treatment of chronic ruptures (>3 months old) have better outcomes compared to non-operative treatment as well. 1,2
Answer C.
References
- Chen, Jie MD, MPH; Kadakia, Rishin MD; Akoh, Craig Chike MD; Schweitzer, Karl Martin Jr MD. Management of Anterior Tibialis Tendon Ruptures. Journal of the American Academy of Orthopaedic Surgeons 29(16):p 691-701, August 15, 2021. | DOI: 10.5435/JAAOS-D-20-00802
- Comadoll S, Ray L, Overman K, Yoon P. Delayed Bilateral Tibialis Anterior Tendon Repair in an Older Man A Case Report. JBJS Case Connect, 13(1):e22.00468 | Case Reports | March 22, 2023
Incorrect
An anterior tibialis tendon rupture is an uncommon injury and is often missed initially due to the continued function of the ankle after the injury. The anterior tibialis tendon supplies 80% of ankle dorsiflexion strength so a tear will cause significant weakness. The ankle is able to dorsiflex after an anterior tibialis tendon rupture due to intact adjacent tendons including the extensor hallucis longus (EHL) and extensor digitorum longus (EDL) tendons. Symptoms of a rupture include a toe dragging gait due to dorsiflexion weakness, a mass on the anterior ankle (retracted tendon stump), and loss of anterior tibial tendon contour. Surgical treatment is recommended in most cases unless contraindicated (low demand patients and medical comorbidities). An ankle foot orthosis is a good option for patients who refuse or aren’t candidates for surgery. A primary repair can usually be performed within the first 6 weeks of injury, however a later presentation usually requires a tendon graft reconstruction or local tendon transfer. Tendon transfers are generally reserved for patients who can’t have a primary repair or graft reconstruction. Surgical treatment of chronic ruptures (>3 months old) have better outcomes compared to non-operative treatment as well. 1,2
Answer C.
References
- Chen, Jie MD, MPH; Kadakia, Rishin MD; Akoh, Craig Chike MD; Schweitzer, Karl Martin Jr MD. Management of Anterior Tibialis Tendon Ruptures. Journal of the American Academy of Orthopaedic Surgeons 29(16):p 691-701, August 15, 2021. | DOI: 10.5435/JAAOS-D-20-00802
- Comadoll S, Ray L, Overman K, Yoon P. Delayed Bilateral Tibialis Anterior Tendon Repair in an Older Man A Case Report. JBJS Case Connect, 13(1):e22.00468 | Case Reports | March 22, 2023
-
Question 4 of 55
4. Question
A 56 year old male presents to your office with two months of lower back pain. He denies a known injury or precipitating event. Over the last few weeks he has developed right sided leg pain that radiates to his calf. On physical exam his light touch sensation and motor strength are intact in bilateral lower extremities. Sagittal and axial MRI images (figures 1 and 2) show multilevel spondylosis, epidural lipomatosis present from L3-SI, and congenitally short pedicles. The patients BMI is 22 and he doesn’t take any prescription medications, although he hasn’t seen a primary care physician in several years. What is the next best step in treatment?
Correct
Spinal epidural lipomatosis is a rare condition characterized by overgrowth of adipose tissue within the spinal canal. The etiology is unclear but can be more common in patients with exogenous steroid use, obesity, endogenous steroid hormonal disease, and prior spinal surgery. Progression of epidural lipomatosis leads to compression of the surrounding neural structures which potentially causes neurological deficits. Symptoms correlate with the severity of the nerve compression and location of the lipomatosis. Symptoms can range from radiculopathy, to myelopathy, and paraplegia. Treatment involves addressing the underlying cause including tapering off, or discontinuing exogenous steroid use, treating hormonal disorders, and weight loss. Approximately 17% of patients with epidural lipomatosis will have idiopathic disease, which can be established only when an endocrinology work-up rules out endogenous steroid production. Hormonal diseases such as hypothyroidism, Cushing syndrome, carcinoid tumor, and pituitary prolactinoma have been associated with epidural lipomatosis. Decompressive surgery is recommended if conservative treatments have failed and can be a very effective treatment option. 1,2
Answer C.
References
- Fassett DR, Schmidt MH. Spinal epidural lipomatosis: a review of its causes and recommendations for treatment. Neurosurgical focus. 2004 Apr 1;16(4):1-3.
- Walker PB, Sark C, Brennan G, Smith T, Sherman WF, Kaye AD. Spinal epidural lipomatosis: a comprehensive review. Orthopedic Reviews. 2021;13(2).
Incorrect
Spinal epidural lipomatosis is a rare condition characterized by overgrowth of adipose tissue within the spinal canal. The etiology is unclear but can be more common in patients with exogenous steroid use, obesity, endogenous steroid hormonal disease, and prior spinal surgery. Progression of epidural lipomatosis leads to compression of the surrounding neural structures which potentially causes neurological deficits. Symptoms correlate with the severity of the nerve compression and location of the lipomatosis. Symptoms can range from radiculopathy, to myelopathy, and paraplegia. Treatment involves addressing the underlying cause including tapering off, or discontinuing exogenous steroid use, treating hormonal disorders, and weight loss. Approximately 17% of patients with epidural lipomatosis will have idiopathic disease, which can be established only when an endocrinology work-up rules out endogenous steroid production. Hormonal diseases such as hypothyroidism, Cushing syndrome, carcinoid tumor, and pituitary prolactinoma have been associated with epidural lipomatosis. Decompressive surgery is recommended if conservative treatments have failed and can be a very effective treatment option. 1,2
Answer C.
References
- Fassett DR, Schmidt MH. Spinal epidural lipomatosis: a review of its causes and recommendations for treatment. Neurosurgical focus. 2004 Apr 1;16(4):1-3.
- Walker PB, Sark C, Brennan G, Smith T, Sherman WF, Kaye AD. Spinal epidural lipomatosis: a comprehensive review. Orthopedic Reviews. 2021;13(2).
-
Question 5 of 55
5. Question
A 28 year old male presents to your office with a right index finger mass for a few months. He is a plumber and he noticed his right index finger comes in contact with plumbing parts throughout the day. The mass on his knuckle has grown slowly and is non-painful, but he is concerned that it may be cancerous. On physical exam he has a firm skin colored nodule at the proximal interphalangeal joint of the right index finger. He has full range of motion of the finger without pain. Photos of the finger are shown in figures 1 and 2. X-rays of the finger do not show any signs of fracture or arthritis. What is the next best step in treatment?
Correct
Knuckle pads, also known as “Garrod’s nodes” are firm skin colored subcutaneous nodules generally located at the proximal interphalangeal joints. The cause of knuckle pads is unknown but they are generally painless and can be associated with repetitive friction and trauma to the hands. The most common age group for these nodes to occur in is between15 to 30 years of age with an equal sex distribution. On physical exam the nodes feel like a non-compressible mass and the adjacent tendons and joints are normal. Similar growths in the fingers include Bouchard and Heberdern’s nodes, rheumatoid nodules, cysts, and tumors. If the diagnosis is unclear based on the clinical picture, an ultrasound, MRI, or biopsy can be obtained. It is important to differentiate finger tumors on exam to prevent an expensive work-up and for patient reassurance. Knuckle pads are generally only a cosmetic deformity and treatment is conservative with avoidance of repetitive trauma if possible. 1,2
Answer A.
References
- Tamborrini G, Gengenbacher M, Bianchi S. Knuckle pads – a rare finding. J Ultrason. 2012 Dec;12(51):493-8. doi: 10.15557/JoU.2012.0037. Epub 2012 Dec 30. PMID: 26672439; PMCID: PMC4603232.
- Vincek V, Vause A, Vincek E, Marion T, Bender N, Carstens S, Motaparthi K. Knuckle pads: an ancient disease frequently misdiagnosed because of minimal modern attention. Dermatology Online Journal. 2021;27(9).
Incorrect
Knuckle pads, also known as “Garrod’s nodes” are firm skin colored subcutaneous nodules generally located at the proximal interphalangeal joints. The cause of knuckle pads is unknown but they are generally painless and can be associated with repetitive friction and trauma to the hands. The most common age group for these nodes to occur in is between15 to 30 years of age with an equal sex distribution. On physical exam the nodes feel like a non-compressible mass and the adjacent tendons and joints are normal. Similar growths in the fingers include Bouchard and Heberdern’s nodes, rheumatoid nodules, cysts, and tumors. If the diagnosis is unclear based on the clinical picture, an ultrasound, MRI, or biopsy can be obtained. It is important to differentiate finger tumors on exam to prevent an expensive work-up and for patient reassurance. Knuckle pads are generally only a cosmetic deformity and treatment is conservative with avoidance of repetitive trauma if possible. 1,2
Answer A.
References
- Tamborrini G, Gengenbacher M, Bianchi S. Knuckle pads – a rare finding. J Ultrason. 2012 Dec;12(51):493-8. doi: 10.15557/JoU.2012.0037. Epub 2012 Dec 30. PMID: 26672439; PMCID: PMC4603232.
- Vincek V, Vause A, Vincek E, Marion T, Bender N, Carstens S, Motaparthi K. Knuckle pads: an ancient disease frequently misdiagnosed because of minimal modern attention. Dermatology Online Journal. 2021;27(9).
-
Question 6 of 55
6. Question
A 42 year old male presents to the emergency room with severe right ankle and knee pain after falling while skating on the ice. He was tripped from behind and his ankle buckled from under him. He had immediate pain and was unable to bear weight. AP and lateral x-rays of the ankle show there to be subtle syndesmotic widening and a posterior malleolar fracture (Figures 1 and 2). AP and lateral x-rays of the knee show a proximal fibular fracture (Figures 3 and 4). What is this fracture pattern called?
Correct
A Maisonneauve fracture is a fracture of the proximal fibula with associated ankle syndesmotic injury. The mechanism is an external rotation of the ankle which causes rupture of the anterior tibiofibular ligament (ATFL) and the interosseous tibiofibular (syndesmosis) ligament. Proximal fibular fractures are often missed with this injury pattern which is often mistaken for an ankle sprain. Maisonneauve fractures are almost always unstable and require surgical fixation with a closed reduction and fixation of the syndesmosis. A Monteggia fracture is a fracture of the proximal ulna with an associated dislocation of the radial head. The Galeazzi fracture is a fracture of the middle to distal one-third of the radius with an associated dislocation or subluxation of the distal radioulnar joint (DRUJ). A Jones fracture is a fracture of the 5th metatarsal at the proximal diaphyseal-metaphyseal junction. 1,2
Answer D.
References
- Bartoníček J, Rammelt S, Tuček M. Maisonneuve Fractures of the Ankle: A Critical Analysis Review. JBJS Rev. 2022 Feb 21;10(2). doi: 10.2106/JBJS.RVW.21.00160. PMID: 35180143.
- Sproule JA, Khalid M, O’Sullivan M, McCabe JP. Outcome after surgery for Maisonneuve fracture of the fibula. Injury. 2004 Aug 1;35(8):791-8.
Incorrect
A Maisonneauve fracture is a fracture of the proximal fibula with associated ankle syndesmotic injury. The mechanism is an external rotation of the ankle which causes rupture of the anterior tibiofibular ligament (ATFL) and the interosseous tibiofibular (syndesmosis) ligament. Proximal fibular fractures are often missed with this injury pattern which is often mistaken for an ankle sprain. Maisonneauve fractures are almost always unstable and require surgical fixation with a closed reduction and fixation of the syndesmosis. A Monteggia fracture is a fracture of the proximal ulna with an associated dislocation of the radial head. The Galeazzi fracture is a fracture of the middle to distal one-third of the radius with an associated dislocation or subluxation of the distal radioulnar joint (DRUJ). A Jones fracture is a fracture of the 5th metatarsal at the proximal diaphyseal-metaphyseal junction. 1,2
Answer D.
References
- Bartoníček J, Rammelt S, Tuček M. Maisonneuve Fractures of the Ankle: A Critical Analysis Review. JBJS Rev. 2022 Feb 21;10(2). doi: 10.2106/JBJS.RVW.21.00160. PMID: 35180143.
- Sproule JA, Khalid M, O’Sullivan M, McCabe JP. Outcome after surgery for Maisonneuve fracture of the fibula. Injury. 2004 Aug 1;35(8):791-8.
-
Question 7 of 55
7. Question
A 56 year old male presents to your office with a right middle finger injury that occurred one month ago and has progressively become deformed. Last month his finger hit a sharp edge of a fan and he sustained a laceration to the dorsum of the proximal interphalangeal joint. He originally went to a local urgent care for sutures and wound management but over the last few weeks the finger has become deformed. Pictures of the finger are shown in figures 1 and 2. On physical exam he has a healing laceration to the dorsum of the PIP joint. The PIP joint is hyperflexed and the DIP joint is in hyperextension consistent with a Boutonniere deformity. What is the best treatment option?
Correct
The patient sustained a traumatic boutonnière deformity which consists of flexion of the proximal interphalangeal (PIP) joint and hyperextension of the distal interphalangeal (DIP) joint. The laceration disrupted the central slip over the PIP joint. The anatomy that contributes to finger extension is complex but in general the deformity is attributed to disruption of the central slip and triangular ligament on the dorsum of the finger with subsequent subluxation of the lateral bands. The deformity may not become evident until days or weeks after the injury. The Elson test is the most reliable way to diagnosis a central slip injury before a deformity is present. Treatment is usually nonoperative with placing the PIP joint in extension for 4-8 weeks to allow the central slip to heal. Range of motion is allowed at the DIP joint. Patients with an open laceration to the dorsum of the PIP joint may benefit from early irrigation and debridement with primary repair of the central slip. 1,2
Answer A.
References
- Image Quiz Traumatic Boutonniere Deformity. JBJS JOPA, 7(3)e0005 Image Quiz September 19, 2019
- McKeon KE, Lee DH. Posttraumatic boutonnière and swan neck deformities. JAAOS-Journal of the American Academy of Orthopaedic Surgeons. 2015 Oct 1;23(10):623-32.
Incorrect
The patient sustained a traumatic boutonnière deformity which consists of flexion of the proximal interphalangeal (PIP) joint and hyperextension of the distal interphalangeal (DIP) joint. The laceration disrupted the central slip over the PIP joint. The anatomy that contributes to finger extension is complex but in general the deformity is attributed to disruption of the central slip and triangular ligament on the dorsum of the finger with subsequent subluxation of the lateral bands. The deformity may not become evident until days or weeks after the injury. The Elson test is the most reliable way to diagnosis a central slip injury before a deformity is present. Treatment is usually nonoperative with placing the PIP joint in extension for 4-8 weeks to allow the central slip to heal. Range of motion is allowed at the DIP joint. Patients with an open laceration to the dorsum of the PIP joint may benefit from early irrigation and debridement with primary repair of the central slip. 1,2
Answer A.
References
- Image Quiz Traumatic Boutonniere Deformity. JBJS JOPA, 7(3)e0005 Image Quiz September 19, 2019
- McKeon KE, Lee DH. Posttraumatic boutonnière and swan neck deformities. JAAOS-Journal of the American Academy of Orthopaedic Surgeons. 2015 Oct 1;23(10):623-32.
-
Question 8 of 55
8. Question
A 45 year old male presents to your office with severe right hip pain for 3 months. He denies having a known injury or precipitating event. Initial x-rays taken in the office 2 months ago show mild arthritis but no evidence of a fracture (Figures 1 and 2). MRI of the hip shows subarticular edema in the femoral head extending into the femoral neck (figure 3). He had a left hip replacement 4 years ago for a spontaneous displaced femoral neck fracture. A recent vitamin D level is low at 15 ng/ml and bone mineral density at the left femoral neck is T score -1.9. On physical exam he has pain with weight bearing of the right hip which is made worse with passive flexion and internal rotation. The patient denies alcohol abuse, previous steroid use, and history of trauma. You suspect transient osteoporosis of the femoral neck. What is the best treatment option?
Correct
The patients presentation is concerning for transient osteoporosis of the hip (TOH). TOH is characterized by an acute onset of hip pain in middle aged men and classically in women in their third trimester. The pain can be severe and disabling and is not associated with a history of trauma. TOH is a temporary osteoporosis of the proximal femur which can also be seen in malnutrition and pregnancy. TOH can be difficult to distinguish between early stage osteonecrosis and subchondral insufficiency fractures and generally an MRI is the most helpful diagnostic study to differentiate. TOH is characterized by diffuse bone marrow edema on MRI, and unlike most cases of osteonecrosis, has intact subchondral bone. TO is self limiting and symptoms usually resolve after a period of 6-8 months. Patients can be treated with weight bearing restrictions and NSAIDs to manage symptoms and calcium and vitamin D supplements to help improve bone quality. 1,2
Answer A.
References
- Korompilias, Anastasios V. MD; Karantanas, Apostolos H. MD; Lykissas, Marios G. MD; Beris, Alexandros E. MD. Transient Osteoporosis. Journal of the American Academy of Orthopaedic Surgeons 16(8):p 480-489, August 2008.
- Vogler J. Caracciolo J, Cheong D. Bilateral Transient Osteoporosis of the Hip. JBJS Case Connect, 4(3):e56 | Case Reports | July 09, 2014
Incorrect
The patients presentation is concerning for transient osteoporosis of the hip (TOH). TOH is characterized by an acute onset of hip pain in middle aged men and classically in women in their third trimester. The pain can be severe and disabling and is not associated with a history of trauma. TOH is a temporary osteoporosis of the proximal femur which can also be seen in malnutrition and pregnancy. TOH can be difficult to distinguish between early stage osteonecrosis and subchondral insufficiency fractures and generally an MRI is the most helpful diagnostic study to differentiate. TOH is characterized by diffuse bone marrow edema on MRI, and unlike most cases of osteonecrosis, has intact subchondral bone. TO is self limiting and symptoms usually resolve after a period of 6-8 months. Patients can be treated with weight bearing restrictions and NSAIDs to manage symptoms and calcium and vitamin D supplements to help improve bone quality. 1,2
Answer A.
References
- Korompilias, Anastasios V. MD; Karantanas, Apostolos H. MD; Lykissas, Marios G. MD; Beris, Alexandros E. MD. Transient Osteoporosis. Journal of the American Academy of Orthopaedic Surgeons 16(8):p 480-489, August 2008.
- Vogler J. Caracciolo J, Cheong D. Bilateral Transient Osteoporosis of the Hip. JBJS Case Connect, 4(3):e56 | Case Reports | July 09, 2014
-
Question 9 of 55
9. Question
A 12 year-old girl presents to your office with left thumb pain after a fall on the playground two days earlier. She admits to falling on the hand and having immediate thumb pain after. On physical exam she has swelling and tenderness to palpation over the distal phalanx. The nail bed and eponychial fold are intact with a mild subungual hematoma. She is able to fully extend the distal phalanx. AP and lateral x-rays are shown in figures 1 and 2. A colleague mentions to you his concern that this could be a Seymour fracture. What diagnostic or exam findings would suggest the patient’s injury is a Seymour fracture?
Correct
Seymour fractures are distal phalanx physeal fractures that result in a laceration to the nail bed which often causes the nail plate to lye superficial to the eponychial fold. The nail bed laceration with associated displaced distal phalanx physeal fracture results in an open fracture that has a risk of infection. Lack of intervention or late presentation can lead to osteomyelitis, growth arrest, and persistent mallet deformity. Physical exam findings may include a slightly flexed distal phalanx, the nail may appear longer due the nail plate being superficial to the eponychial fold, and eschar may be present in the eponychial fold if the injury is seen a few days postinjury. AP radiographs may appear normal and a lateral radiograph will likely show physeal widening. The differential diagnosis includes a pediatric mallet finger which is generally a salter harris III or IV fracture. Most Seymour fractures are open and have soft tissue interposition at the fracture site that will prevent fracture healing. Open fractures require open reduction and pinning across the distal interphalangeal joint with nailbed repair. 1,2
Answer C.
References
- Abzug, Joshua M. MD; Dua, Karan MD; Bauer, Andrea Sesko MD; Cornwall, Roger MD; Wyrick, Theresa O. MD. Pediatric Phalanx Fractures. Journal of the American Academy of Orthopaedic Surgeons 24(11):p e174-e183, November 2016. | DOI: 10.5435/JAAOS-D-16-00199
2. Kiely AL, Nolan GS, Cooper LRL. The optimal management of Seymour fractures in children and adolescents: a systematic review protocol. Syst Rev. 2020 Jun 23;9(1):150. doi: 10.1186/s13643-020-01407-5. PMID: 32576259; PMCID: PMC7313162
Incorrect
Seymour fractures are distal phalanx physeal fractures that result in a laceration to the nail bed which often causes the nail plate to lye superficial to the eponychial fold. The nail bed laceration with associated displaced distal phalanx physeal fracture results in an open fracture that has a risk of infection. Lack of intervention or late presentation can lead to osteomyelitis, growth arrest, and persistent mallet deformity. Physical exam findings may include a slightly flexed distal phalanx, the nail may appear longer due the nail plate being superficial to the eponychial fold, and eschar may be present in the eponychial fold if the injury is seen a few days postinjury. AP radiographs may appear normal and a lateral radiograph will likely show physeal widening. The differential diagnosis includes a pediatric mallet finger which is generally a salter harris III or IV fracture. Most Seymour fractures are open and have soft tissue interposition at the fracture site that will prevent fracture healing. Open fractures require open reduction and pinning across the distal interphalangeal joint with nailbed repair. 1,2
Answer C.
References
- Abzug, Joshua M. MD; Dua, Karan MD; Bauer, Andrea Sesko MD; Cornwall, Roger MD; Wyrick, Theresa O. MD. Pediatric Phalanx Fractures. Journal of the American Academy of Orthopaedic Surgeons 24(11):p e174-e183, November 2016. | DOI: 10.5435/JAAOS-D-16-00199
2. Kiely AL, Nolan GS, Cooper LRL. The optimal management of Seymour fractures in children and adolescents: a systematic review protocol. Syst Rev. 2020 Jun 23;9(1):150. doi: 10.1186/s13643-020-01407-5. PMID: 32576259; PMCID: PMC7313162
-
Question 10 of 55
10. Question
A 39 year old male presents to your office with right shoulder pain for three months. He denies having a known injury or precipitating event. The pain is in the posterior shoulder at the periscapular region. He has tried oral steroids with only minimal relief. The pain is not associated with shoulder motion and seems to be relentless at times, even at rest. On physical exam he has a positive Spurling’s test with radiation to the right periscapular region. He has negative shoulder impingement signs and active range of motion of the shoulder is without pain or weakness. X-rays taken in the office do not show there to be any arthritic changes or abnormalities. Sagittal MRI of the cervical spine is shown in figure 1. Which choice is the best treatment option for this patient?
Correct
The patient is presenting with a C3-C4 cervical disk herniation causing cervical radiculopathy in a C4 distribution. Cervical radiculopathy is self-limiting and 75% of patients will have resolution of symptoms in 2-3 months with non-operative treatment. A period of rest, physical therapy with cervical traction, and oral steroids help improve symptoms. If symptoms persist beyond 1-2 months and patients have significant intolerable pain then cervical steroid injections can help provide relief. Interlaminar and transforaminal are the two approaches used to administer cervical epidural injections for relief of cervical radiculopathy. Interlaminar injections are technically easier to administer than transforaminal injections and are more commonly used. Both injections provide pain relief in up to 72% of patients at two weeks and clinical benefits start to subside significantly after 8 weeks. Typically, the highest level that can be safely injected with an interlaminar approach is the C6-C7 level were the epidural space is widest. The dorsal epidural space is narrower at higher levels so risk of intrathecal needle penetration increases as you move up the cervical spine. 1,2
Answer D.
References
1. Rhee, John M. MD; Yoon, Tim MD; Riew, K. Daniel MD. Cervical Radiculopathy. Journal of the American Academy of Orthopaedic Surgeons 15(8):p 486-494, August 2007.
2. Lee, Jung Hwan MD, PhD; Lee, Sang-Ho MD, PhD. Comparison of Clinical Efficacy Between Interlaminar and Transforaminal Epidural Injection in Patients With Axial Pain due to Cervical Disc Herniation. Medicine 95(4):p e2568, January 2016.Incorrect
The patient is presenting with a C3-C4 cervical disk herniation causing cervical radiculopathy in a C4 distribution. Cervical radiculopathy is self-limiting and 75% of patients will have resolution of symptoms in 2-3 months with non-operative treatment. A period of rest, physical therapy with cervical traction, and oral steroids help improve symptoms. If symptoms persist beyond 1-2 months and patients have significant intolerable pain then cervical steroid injections can help provide relief. Interlaminar and transforaminal are the two approaches used to administer cervical epidural injections for relief of cervical radiculopathy. Interlaminar injections are technically easier to administer than transforaminal injections and are more commonly used. Both injections provide pain relief in up to 72% of patients at two weeks and clinical benefits start to subside significantly after 8 weeks. Typically, the highest level that can be safely injected with an interlaminar approach is the C6-C7 level were the epidural space is widest. The dorsal epidural space is narrower at higher levels so risk of intrathecal needle penetration increases as you move up the cervical spine. 1,2
Answer D.
References
1. Rhee, John M. MD; Yoon, Tim MD; Riew, K. Daniel MD. Cervical Radiculopathy. Journal of the American Academy of Orthopaedic Surgeons 15(8):p 486-494, August 2007.
2. Lee, Jung Hwan MD, PhD; Lee, Sang-Ho MD, PhD. Comparison of Clinical Efficacy Between Interlaminar and Transforaminal Epidural Injection in Patients With Axial Pain due to Cervical Disc Herniation. Medicine 95(4):p e2568, January 2016. -
Question 11 of 55
11. Question
A 43 year old male presents to your office with neck pain for over 10 years. He has had a 20 year history of working in the military carrying very heavy things and wearing helmets and he thinks that this may have played a role in worsening neck pain. He describes intermittent headaches and neck pain over the center of his cervical spine that is made worse with range of motion and prolonged positioning such when he is driving. He denies any pain radiating down the arms or weakness in the upper extremities. He has used massage therapy, anti-inflammatories, and muscle relaxers for temporary relief. This pain can be 8 out of 10 on the numeric pain scale and affects his ability to perform range of motion of the neck with activities of daily living. AP and lateral x-rays of the cervical spine show mild facet arthritis (figures 1 and 2). MRI of the neck shows mild facet inflammation but no evidence of nerve impingement. What is the next best step in treatment?
Correct
Cervical facet joint arthritis is a common cause of axial (non-radiating) neck pain. The cervical facet joints are supplied by an abundant nerve supply and are a frequent source of neck pain. The differential diagnosis of axial neck pain is broad which makes treatment of this extremely common complaint so difficult. Neck pain symptoms that point to a facet joint origin include pain with prolong positioning (such as hair dressers and dentists), pain made worse with neck extension and lateral rotation, and occipital headaches. The facet joints from C3 to C7 are innervated by the medial branch of the ramis dorsalis of the segmental nerve. Physical exam findings with facet joint arthritis will include pain to palpation over the facet joints, pain with neck rotation, and an absence of motor weakness and sensory deficits of the upper extremities. To confirm the diagnosis of facet joint pain medial branch blocks (MBBs) can be performed for diagnostic and therapeutic purposes. MBBs are performed under fluoroscopic guidance and an anesthetic such as lidocaine or marcaine is injected with or without a corticosteroid around the facet joint capsule where the nerves are located. If patients get significant (>50%) relief from the MBB than facet joint etiology can be confirmed. Radiofrequency ablation (RFA) is another treatment option that can provide longer lasting benefits compared to MBBs for facet joint pain, but often insurance carriers require documentation of at least 1-2 successful MBBs before approving the RFA. 1,2
Answer D.
References
- Van Eerd M, Patijn J, Lataster A, Rosenquist RW, Van Kleef M, Mekhail N, Van Zundert J. Cervical facet pain. Evidence‐Based Interventional Pain Medicine: According to Clinical Diagnoses. 2011 Dec 2:31-9.
2. Lawson GE, Nolet PS, Little AR, Bhattacharyya A, Wang V, Lawson CA, Ko GD. Medial Branch
Incorrect
Cervical facet joint arthritis is a common cause of axial (non-radiating) neck pain. The cervical facet joints are supplied by an abundant nerve supply and are a frequent source of neck pain. The differential diagnosis of axial neck pain is broad which makes treatment of this extremely common complaint so difficult. Neck pain symptoms that point to a facet joint origin include pain with prolong positioning (such as hair dressers and dentists), pain made worse with neck extension and lateral rotation, and occipital headaches. The facet joints from C3 to C7 are innervated by the medial branch of the ramis dorsalis of the segmental nerve. Physical exam findings with facet joint arthritis will include pain to palpation over the facet joints, pain with neck rotation, and an absence of motor weakness and sensory deficits of the upper extremities. To confirm the diagnosis of facet joint pain medial branch blocks (MBBs) can be performed for diagnostic and therapeutic purposes. MBBs are performed under fluoroscopic guidance and an anesthetic such as lidocaine or marcaine is injected with or without a corticosteroid around the facet joint capsule where the nerves are located. If patients get significant (>50%) relief from the MBB than facet joint etiology can be confirmed. Radiofrequency ablation (RFA) is another treatment option that can provide longer lasting benefits compared to MBBs for facet joint pain, but often insurance carriers require documentation of at least 1-2 successful MBBs before approving the RFA. 1,2
Answer D.
References
- Van Eerd M, Patijn J, Lataster A, Rosenquist RW, Van Kleef M, Mekhail N, Van Zundert J. Cervical facet pain. Evidence‐Based Interventional Pain Medicine: According to Clinical Diagnoses. 2011 Dec 2:31-9.
2. Lawson GE, Nolet PS, Little AR, Bhattacharyya A, Wang V, Lawson CA, Ko GD. Medial Branch
-
Question 12 of 55
12. Question
A 77 year old female presents to your office with chronic lower back pain for years. She has tried physical therapy and does have a home exercise program which offers some relief. She has tried medications including muscle relaxants and NSAIDs with minimal relief. Her pain does not radiate down her legs and she denies bowel or bladder dysfunction. On physical exam she has pain to palpation over the bilateral lumbar facet joints at L4-5 and L5-S1. The pain is worse with active back extension and lateral bending to her side. AP and lateral x-rays of the lumbar spine show multi-level spondylosis (figures 1 and 2). MRI of the spine confirms significant degenerative changes at the lumbar facet joints. Which treatment option below would offer this patient the longest lasting relief for her lower back pain?
Correct
The lumbar facet joints provide stability to the spine during range of motion, with flexion in particular. The lumbar facets can degenerate and become a cause of chronic lower back pain or an all too common complaint in clinical practice. In fact, facet mediated pain is the cause of up to 30%-45% of patients who present with chronic back pain. Facet joint pain is typically worse in the morning, the lower back pain is made worse with flexion/extension and rotation, and pain to palpation over the facet joints is common. If patients fail treatment with a period of physical therapy and other conservative measures (chiropractor, massage therapy, acupuncture) then spine injections are generally the next step. Medial branch blocks help confirm the diagnosis of facet joint pain when xrays, MRI, and clinical exam all point in that direction. Medial branch blocks place an anesthetic at the facet joint capsule where the tiny nerves innervate the joint. Greater than 50% pain relief after a medial branch block is diagnostic of facet joint inflammation. Once a medial branch block has documented success then patients can be offered a radiofrequency ablation which cauterizes the nerves and offers longer lasting relief. Lumbar RFA offers successful pain relief for up to 6-12 months or more. In comparison, steroid injections into the facet joints offer similar relief to RFA for up to 3-6 months but have less sustained relief thereafter. 1,2
Answer A.
References
- Chavez EM. CLumbar Facet Joint Radiofrequency Ablation Versus Intra-articular Lumbar Facet Joint Injections Using Corticosteroids in the Reduction of Chronic Low Back Pain Related to the Facet Joints: A Systematic Review of Literature. JBJS JOPA, 8(2):e19.00033
- McCormick ZL, Marshall B, Walker J, McCarthy R, Walega DR. Long-Term Function, Pain and Medication Use Outcomes of Radiofrequency Ablation for Lumbar Facet Syndrome. Int J Anesth Anesth. 2015;2(2):028.
Incorrect
The lumbar facet joints provide stability to the spine during range of motion, with flexion in particular. The lumbar facets can degenerate and become a cause of chronic lower back pain or an all too common complaint in clinical practice. In fact, facet mediated pain is the cause of up to 30%-45% of patients who present with chronic back pain. Facet joint pain is typically worse in the morning, the lower back pain is made worse with flexion/extension and rotation, and pain to palpation over the facet joints is common. If patients fail treatment with a period of physical therapy and other conservative measures (chiropractor, massage therapy, acupuncture) then spine injections are generally the next step. Medial branch blocks help confirm the diagnosis of facet joint pain when xrays, MRI, and clinical exam all point in that direction. Medial branch blocks place an anesthetic at the facet joint capsule where the tiny nerves innervate the joint. Greater than 50% pain relief after a medial branch block is diagnostic of facet joint inflammation. Once a medial branch block has documented success then patients can be offered a radiofrequency ablation which cauterizes the nerves and offers longer lasting relief. Lumbar RFA offers successful pain relief for up to 6-12 months or more. In comparison, steroid injections into the facet joints offer similar relief to RFA for up to 3-6 months but have less sustained relief thereafter. 1,2
Answer A.
References
- Chavez EM. CLumbar Facet Joint Radiofrequency Ablation Versus Intra-articular Lumbar Facet Joint Injections Using Corticosteroids in the Reduction of Chronic Low Back Pain Related to the Facet Joints: A Systematic Review of Literature. JBJS JOPA, 8(2):e19.00033
- McCormick ZL, Marshall B, Walker J, McCarthy R, Walega DR. Long-Term Function, Pain and Medication Use Outcomes of Radiofrequency Ablation for Lumbar Facet Syndrome. Int J Anesth Anesth. 2015;2(2):028.
-
Question 13 of 55
13. Question
A 46 year old Spanish speaking gentlemen presented to urgent care two weeks ago after a fall while working on a construction site. He was framing a house when he lost his balance and fell awkwardly off a platform on the right knee and left wrist. The wrist and right knee caused him significant pain which prompted him to present to an urgent care clinic. X-rays of the wrist were taken at the clinic and were negative for a fracture. He did not have xrays of the knee at his initial visit. He was placed in a wrist splint and hinged knee brace to the right knee with instructions to weight bear as tolerated and follow-up with orthopedics. He now presents for orthopedic follow up with increasing right knee pain and instability. He denies having a previous knee injury and any current numbness to the right leg. On physical exam he has gross instability about his knee with significant gapping medially with valgus stress. He has positive anterior drawer, posterior drawer, and Lachman tests. He does have palpable posterior tibialis and dorsal pedialis pulses and he has good strength with dorsiflexion and plantar flexion of the toes and ankle. He is able to do a straight leg raise on the right knee. AP and lateral view xrays of the right knee are shown in figures 1 and 2. What is the best treatment option at this point?
Correct
Multiligament knee injuries have a strong association with knee dislocations and neurovascular injuries so early recognition and treatment is critical in preventing disabling complications. Knee instability caused by tears of the anterior and posterior cruciate ligaments and collateral ligaments can cause injury to the popliteal artery, peroneal nerve, and other soft tissues of the knee (particularly the meniscus). The initial work-up for any suspected knee injury is an xray which is generally normal. Knee dislocations can spontaneously reduce so a normal xray never rules out a significant knee injury. A stress xray with a varus stress place on the knee is a dynamic test for a posterior corner injury whereas a valgus stress test is used to determine the integrity of the medial collateral ligament. MRI is the gold standard study for establishing the diagnosis and for preoperative planning. The initial treatment for a patient with a suspected multiligament injury is non-weightbearing on the injured knee. Weight bearing on the injured extremity could result in a dislocation event and neurovascular injury. A vascular injury is a devastating consequence of knee instability as it increases the probability of compartment syndrome and knee amputation. Treatment generally involves surgical reconstruction of the ACL, PCL, and or the MCL in an acute or delayed fashion (with no clear difference in outcomes between the two). 1,2
Answer C.
References
- Burrus M, Werner B, Griffin J, Gwathmey F, Winston M, Miller M. Diagnostic and Management Strategies for Multiligament Knee Injuries. A Critical Analysis Review. JBJS Rev, 4(2):e1 | Review Article | February 02, 2016
- Fortier LM, Stylli JA, Civilette M, Duran NS, Hanukaai S, Wilder H, Sherman WF, Kaye AD. An evidence-based approach to multi-ligamentous knee injuries. Orthopedic Reviews. 2022;14(3).
Incorrect
Multiligament knee injuries have a strong association with knee dislocations and neurovascular injuries so early recognition and treatment is critical in preventing disabling complications. Knee instability caused by tears of the anterior and posterior cruciate ligaments and collateral ligaments can cause injury to the popliteal artery, peroneal nerve, and other soft tissues of the knee (particularly the meniscus). The initial work-up for any suspected knee injury is an xray which is generally normal. Knee dislocations can spontaneously reduce so a normal xray never rules out a significant knee injury. A stress xray with a varus stress place on the knee is a dynamic test for a posterior corner injury whereas a valgus stress test is used to determine the integrity of the medial collateral ligament. MRI is the gold standard study for establishing the diagnosis and for preoperative planning. The initial treatment for a patient with a suspected multiligament injury is non-weightbearing on the injured knee. Weight bearing on the injured extremity could result in a dislocation event and neurovascular injury. A vascular injury is a devastating consequence of knee instability as it increases the probability of compartment syndrome and knee amputation. Treatment generally involves surgical reconstruction of the ACL, PCL, and or the MCL in an acute or delayed fashion (with no clear difference in outcomes between the two). 1,2
Answer C.
References
- Burrus M, Werner B, Griffin J, Gwathmey F, Winston M, Miller M. Diagnostic and Management Strategies for Multiligament Knee Injuries. A Critical Analysis Review. JBJS Rev, 4(2):e1 | Review Article | February 02, 2016
- Fortier LM, Stylli JA, Civilette M, Duran NS, Hanukaai S, Wilder H, Sherman WF, Kaye AD. An evidence-based approach to multi-ligamentous knee injuries. Orthopedic Reviews. 2022;14(3).
-
Question 14 of 55
14. Question
A 72 year old female with a history of rheumatoid arthritis presents to your office with an inability to extend the small and ringer fingers for the past 2 months. She denies a recent injury but her fingers have been deformed for “years” from her RA. On physical exam she has a 60-degree extensor lag to the small finger and an approximately 40-degree extensor lag to the ring finger. She has excellent extension to the index and long fingers. X-rays of the wrist show advanced arthritis to the distal radial and ulnar joint as well as the radiocarpal and mid carpal joints consistent with her rheumatoid arthritis (figures 1 and 2). What is the best treatment option for her loss of finger extension?
Correct
Rheumatoid arthritis is a disease of chronic inflammation which has a predilection to affect the hands. The tendon sheaths of the hand and wrist are prone to inflammation (tenosynovitis) in patients with RA and chronic inflammation can result in tendon deterioration and eventual rupture. Tendon attrition can occur as the tendon glides over prominent joints of RA which further increases the likelihood of rupture. Patients with RA have a 4% incidence of extensor tendon rupture but often never notice the rupture due to baseline dysfunction of their hands. Ideally tenosynovitis can be treated with anti-inflammatories and rest or surgical tenosynovectomy before the tendons rupture. Once the tendons rupture a tendon transfer (using non-affected tendons) or free tendon grafting can help restore finger extension. Primary repair of the ruptured tendons is usually not possible due to the poor quality of the tendon. 1,2
Answer D.
References
- Biehl C, Rupp M, Kern S, Heiss C, ElKhassawna T, Szalay G. Extensor tendon ruptures in rheumatoid wrists. Eur J Orthop Surg Traumatol. 2020 Dec;30(8):1499-1504. doi: 10.1007/s00590-020-02731-1. Epub 2020 Jul 3. PMID: 32621142; PMCID: PMC7680316.
- Williamson L, Mowat A, Burge P. Screening for extensor tendon rupture in rheumatoid arthritis. Rheumatology (Oxford). 2001 Apr;40(4):420-3. doi: 10.1093/rheumatology/40.4.420. PMID: 11312381.
Incorrect
Rheumatoid arthritis is a disease of chronic inflammation which has a predilection to affect the hands. The tendon sheaths of the hand and wrist are prone to inflammation (tenosynovitis) in patients with RA and chronic inflammation can result in tendon deterioration and eventual rupture. Tendon attrition can occur as the tendon glides over prominent joints of RA which further increases the likelihood of rupture. Patients with RA have a 4% incidence of extensor tendon rupture but often never notice the rupture due to baseline dysfunction of their hands. Ideally tenosynovitis can be treated with anti-inflammatories and rest or surgical tenosynovectomy before the tendons rupture. Once the tendons rupture a tendon transfer (using non-affected tendons) or free tendon grafting can help restore finger extension. Primary repair of the ruptured tendons is usually not possible due to the poor quality of the tendon. 1,2
Answer D.
References
- Biehl C, Rupp M, Kern S, Heiss C, ElKhassawna T, Szalay G. Extensor tendon ruptures in rheumatoid wrists. Eur J Orthop Surg Traumatol. 2020 Dec;30(8):1499-1504. doi: 10.1007/s00590-020-02731-1. Epub 2020 Jul 3. PMID: 32621142; PMCID: PMC7680316.
- Williamson L, Mowat A, Burge P. Screening for extensor tendon rupture in rheumatoid arthritis. Rheumatology (Oxford). 2001 Apr;40(4):420-3. doi: 10.1093/rheumatology/40.4.420. PMID: 11312381.
-
Question 15 of 55
15. Question
A 43 year old male presents to your office with chronic right shoulder pain. He has a complicated history with two failed rotator cuff surgeries, the most recent being 6 months ago. An MRI done last month showed there to be a full thickness supraspinatus and infraspinatus tear with tendon retraction and fatty infiltration of the supraspinatus muscle. He has slight superior migration to the humeral head without evidence of glenohumeral arthritis. Figure 1 shows significant fatty infiltration of the supraspinatus tendon on sagittal MRI image. The subscapularis tendon is intact. Which treatment below is the best option for this patient’s posterior superior irreparable rotator cuff tear?
Correct
Rotator cuff pathology is one of the most common causes of shoulder pain which frequently requires surgical intervention. A full thickness rotator cuff tear will not repair itself and therefore requires surgical repair to suture the tendon back to its footprint on the proximal humerus. When the tendon is torn for a long period of time the tendon can retract and become irreparable. Signs of an irreparable rotator cuff tear include tendon retraction to the level of the glenoid or beyond and fatty infiltration of the rotator cuff muscle (disuse of the muscle causes fatty replacement of the muscle fibers as seen on MRI). Treatment options for an irreparable rotator cuff tear include a reverse total shoulder arthroplasty, tendon transfers, bridging interposition grafts, superior capsular reconstruction, and subacromial spacers. In young athletic patients, arthroplasty is not a good option as the function of an arthroplasty is limited and will likely require a revision if done in younger patients. Tendon transfers, such as lower trapezius and latissimus transfers, are ideal for posterior superior tears with an intact subscapularis tendon. Between the two tendon transfer options, a lower trapezius transfer has been shown restore native glenohumeral biomechanics and external rotation strength to a greater degree compared to a latissimus transfer. 1,2
Answer D.
References
- Elhassan, Bassem T. MD; Cox, Ryan M. BS; Shukla, Dave R. MD; Lee, Julia MD; Murthi, Anand M. MD; Tashjian, Robert Z. MD; Abboud, Joseph A. MD. Management of Failed Rotator Cuff Repair in Young Patients. Journal of the American Academy of Orthopaedic Surgeons 25(11):p e261-e271, November 2017. | DOI: 10.5435/JAAOS-D-17-00086
- Omid R, Heckmann N, Wang L, McGarry MH, Vangsness Jr CT, Lee TQ. Biomechanical comparison between the trapezius transfer and latissimus transfer for irreparable posterosuperior rotator cuff tears. Journal of Shoulder and Elbow Surgery. 2015 Oct 1;24(10):1635-43.
Incorrect
Rotator cuff pathology is one of the most common causes of shoulder pain which frequently requires surgical intervention. A full thickness rotator cuff tear will not repair itself and therefore requires surgical repair to suture the tendon back to its footprint on the proximal humerus. When the tendon is torn for a long period of time the tendon can retract and become irreparable. Signs of an irreparable rotator cuff tear include tendon retraction to the level of the glenoid or beyond and fatty infiltration of the rotator cuff muscle (disuse of the muscle causes fatty replacement of the muscle fibers as seen on MRI). Treatment options for an irreparable rotator cuff tear include a reverse total shoulder arthroplasty, tendon transfers, bridging interposition grafts, superior capsular reconstruction, and subacromial spacers. In young athletic patients, arthroplasty is not a good option as the function of an arthroplasty is limited and will likely require a revision if done in younger patients. Tendon transfers, such as lower trapezius and latissimus transfers, are ideal for posterior superior tears with an intact subscapularis tendon. Between the two tendon transfer options, a lower trapezius transfer has been shown restore native glenohumeral biomechanics and external rotation strength to a greater degree compared to a latissimus transfer. 1,2
Answer D.
References
- Elhassan, Bassem T. MD; Cox, Ryan M. BS; Shukla, Dave R. MD; Lee, Julia MD; Murthi, Anand M. MD; Tashjian, Robert Z. MD; Abboud, Joseph A. MD. Management of Failed Rotator Cuff Repair in Young Patients. Journal of the American Academy of Orthopaedic Surgeons 25(11):p e261-e271, November 2017. | DOI: 10.5435/JAAOS-D-17-00086
- Omid R, Heckmann N, Wang L, McGarry MH, Vangsness Jr CT, Lee TQ. Biomechanical comparison between the trapezius transfer and latissimus transfer for irreparable posterosuperior rotator cuff tears. Journal of Shoulder and Elbow Surgery. 2015 Oct 1;24(10):1635-43.
-
Question 16 of 55
16. Question
A 26 year old butcher presents to your office after a thumb laceration while cutting meat at work two weeks ago. The laceration occurred just distal to the interphalangeal joint of the thumb. He was originally seen at an urgent care facility and the wound was irrigated and sutured and he was instructed to follow up as needed after the sutures were removed. However, two weeks after the injury, he noticed the tip of his thumb couldn’t fully extend when he gave his kid a “thumbs up” during a soccer game. On physical exam he cannot extend the distal phalanx of the thumb. He has full extension of the interphalangeal joint. His laceration is well healed. Pictures of the thumb are shown in figures 1 and 2. What is the most likely cause of the patient’s inability to extend the distal phalanx of his right thumb ?
Correct
The extensor pollicis longus (EPL) tendon runs alongside the extensor pollicis brevis tendon and functions to extend the distal phalanx of the thumb. The abductor pollicis brevis and adductor pollicis tendons both attach to the extensor pollicis longus tendon and function to extend the thumbs interphalangeal joint. However, only the EPL extends the distal phalanx of the thumb. The patient’s inability to extend his distal phalanx indicates he likely lacerated his EPL. The diagnosis is generally made clinically and advanced imaging, including MRI, is often unnecessary. Tendon lacerations should be treated acutely, ideally within two weeks of the tendon injury. The lacerated tendon will retract over time which makes an end to end repair more difficult as time passes. Surgical repair is the treatment of choice with an excellent prognosis. Partial tears greater than 50% of the tendon should also be considered for repair to ensure tendon balance and prevent completion of the tear. The EPL is innervated by the posterior interosseus nerve which may be injured at the level of the elbow resulting in weakness of thumb extension. 1,2
Answer B.
References
- Khandwala AR, Blair J, Harris SB, Foster AJ, Elliot D. Immediate repair and early mobilization of the extensor pollicis longus tendon in zones 1 to 4. Journal of Hand Surgery. 2004 Jun;29(3):250-8.
- Khan IA, Varacallo M. Anatomy, Shoulder and Upper Limb, Hand Extensor Pollicis Longus Muscle. [Updated 2022 Dec 21]. In: StatPearls [Internet]. Treasure Island (FL): StatPearls Publishing; 2024 Jan-
Incorrect
The extensor pollicis longus (EPL) tendon runs alongside the extensor pollicis brevis tendon and functions to extend the distal phalanx of the thumb. The abductor pollicis brevis and adductor pollicis tendons both attach to the extensor pollicis longus tendon and function to extend the thumbs interphalangeal joint. However, only the EPL extends the distal phalanx of the thumb. The patient’s inability to extend his distal phalanx indicates he likely lacerated his EPL. The diagnosis is generally made clinically and advanced imaging, including MRI, is often unnecessary. Tendon lacerations should be treated acutely, ideally within two weeks of the tendon injury. The lacerated tendon will retract over time which makes an end to end repair more difficult as time passes. Surgical repair is the treatment of choice with an excellent prognosis. Partial tears greater than 50% of the tendon should also be considered for repair to ensure tendon balance and prevent completion of the tear. The EPL is innervated by the posterior interosseus nerve which may be injured at the level of the elbow resulting in weakness of thumb extension. 1,2
Answer B.
References
- Khandwala AR, Blair J, Harris SB, Foster AJ, Elliot D. Immediate repair and early mobilization of the extensor pollicis longus tendon in zones 1 to 4. Journal of Hand Surgery. 2004 Jun;29(3):250-8.
- Khan IA, Varacallo M. Anatomy, Shoulder and Upper Limb, Hand Extensor Pollicis Longus Muscle. [Updated 2022 Dec 21]. In: StatPearls [Internet]. Treasure Island (FL): StatPearls Publishing; 2024 Jan-
-
Question 17 of 55
17. Question
A 26 year old male presents to your office with a large area of swelling along his lateral thigh and buttock on the left side. He says the swelling originally started when he was thrown from his motorcycle 8 months ago and landed directly on his left side. He had a large area of bruising along his lateral leg after the accident and was seen in the emergency room and no fractures were found. The area of swelling has stayed about the same size for the past 4 months. Over the last 4 months the swelling has gone from soft and fluid filled to more of a firm consistency. Pictures of the thigh are shown in figures 1 and 2. What is the next best step in treatment?
Correct
The patient sustained a Morel-Lavallee (ML) Lesion as a result of his motorcycle accident 8 months ago. A ML lesion is characterized by a traumatic separation of the dermis layer from the adjacent fascial layer. The lymphatic and vascular structures are often torn which results in fluid accumulating between the two layers. The injury presents as a large bruise to the injured region, generally the lateral thigh. The diagnosis is generally made clinically with palpation of a compressible fluid collection that persists weeks to months after an injury. The fluid collection is often noticed weeks after the injury which is why the diagnosis is most often delayed. MRI can help determine the size and extent of the fluid collection but is generally not necessary. Treatment is variable and there is no consensus on which lesions need surgery. Lesions less than 50cm3 generally do well with conservative treatment such as compression therapy, aspiration, and percutaneous drain placement (if necessary). Large lesions have a tendency to persist and may require an open debridement with closure. Chronic lesions are likely to have formed a pseudocapsule and generally require open debridement with removal of the capsule and skin closure. 1,2
Answer D.
References
- Scolaro JA, Chao T, Zamorano DP. The Morel-Lavallée lesion: diagnosis and management. JAAOS-Journal of the American Academy of Orthopaedic Surgeons. 2016 Oct 1;24(10):667-72.
- Powers ML, Hatem SF, Sundaram M. Morel-Lavallee lesion. Orthopedics. 2007 Apr;30(4):250, 322-3. doi: 10.3928/01477447-20070401-10. PMID: 17424683.
Incorrect
The patient sustained a Morel-Lavallee (ML) Lesion as a result of his motorcycle accident 8 months ago. A ML lesion is characterized by a traumatic separation of the dermis layer from the adjacent fascial layer. The lymphatic and vascular structures are often torn which results in fluid accumulating between the two layers. The injury presents as a large bruise to the injured region, generally the lateral thigh. The diagnosis is generally made clinically with palpation of a compressible fluid collection that persists weeks to months after an injury. The fluid collection is often noticed weeks after the injury which is why the diagnosis is most often delayed. MRI can help determine the size and extent of the fluid collection but is generally not necessary. Treatment is variable and there is no consensus on which lesions need surgery. Lesions less than 50cm3 generally do well with conservative treatment such as compression therapy, aspiration, and percutaneous drain placement (if necessary). Large lesions have a tendency to persist and may require an open debridement with closure. Chronic lesions are likely to have formed a pseudocapsule and generally require open debridement with removal of the capsule and skin closure. 1,2
Answer D.
References
- Scolaro JA, Chao T, Zamorano DP. The Morel-Lavallée lesion: diagnosis and management. JAAOS-Journal of the American Academy of Orthopaedic Surgeons. 2016 Oct 1;24(10):667-72.
- Powers ML, Hatem SF, Sundaram M. Morel-Lavallee lesion. Orthopedics. 2007 Apr;30(4):250, 322-3. doi: 10.3928/01477447-20070401-10. PMID: 17424683.
-
Question 18 of 55
18. Question
A 22 year old female presents to your office with right knee pain after a fall at work 2 days ago. She was stepping down from a stool when her foot slipped and her right knee buckled awkwardly. She felt a “pop” and had severe pain after. She has had difficulty bearing weight since the injury. On physician exam she is guarded and very difficult to examine. She has a moderate joint effusion but knee stability is difficult to determine as the patient can not relax her knee. AP x-ray of the right knee is shown in figure 1. What is the most likely diagnosis?
Correct
A Segond fracture is an avulsion fracture of the proximal lateral tibia that is caused by an internal rotation force to the tibia. The fracture is often termed a “lateral capsular sign” and associated with an anterior cruciate ligament (ACL) tear in adults. In patients with a Segond fracture, 95% were found to have abnormalities of the ACL on MRI. Recognition of this fracture pattern is critical to diagnose knee instability which requires an early MRI. A Segond fracture is thought to originate from an avulsion of the anterior lateral ligament, which is still a poorly understood structure in the knee. The avulsion fracture is also in close proximity to the posterior fibers of the iliotibial band and posterior capsule attachments which further confuses the etiology of a Segond fracture. The lateral collateral ligament originates from the femur and inserts into the anterior fibular head, thus an LCL avulsion fracture (which is rare) would not displace from the lateral tibia. 1,2
Answer D.
References
1. Dodds AL, Halewood C, Gupte CM, Williams A, Amis AA. The anterolateral ligament: anatomy, length changes and association with the Segond fracture. The bone & joint journal. 2014 Mar 1;96(3):325-31.
2. Claes S, Luyckx T, Vereecke E, Bellemans J. The Segond fracture: a bony injury of the anterolateral ligament of the knee. Arthroscopy: The Journal of Arthroscopic & Related Surgery. 2014 Nov 1;30(11):1475-82.
Incorrect
A Segond fracture is an avulsion fracture of the proximal lateral tibia that is caused by an internal rotation force to the tibia. The fracture is often termed a “lateral capsular sign” and associated with an anterior cruciate ligament (ACL) tear in adults. In patients with a Segond fracture, 95% were found to have abnormalities of the ACL on MRI. Recognition of this fracture pattern is critical to diagnose knee instability which requires an early MRI. A Segond fracture is thought to originate from an avulsion of the anterior lateral ligament, which is still a poorly understood structure in the knee. The avulsion fracture is also in close proximity to the posterior fibers of the iliotibial band and posterior capsule attachments which further confuses the etiology of a Segond fracture. The lateral collateral ligament originates from the femur and inserts into the anterior fibular head, thus an LCL avulsion fracture (which is rare) would not displace from the lateral tibia. 1,2
Answer D.
References
1. Dodds AL, Halewood C, Gupte CM, Williams A, Amis AA. The anterolateral ligament: anatomy, length changes and association with the Segond fracture. The bone & joint journal. 2014 Mar 1;96(3):325-31.
2. Claes S, Luyckx T, Vereecke E, Bellemans J. The Segond fracture: a bony injury of the anterolateral ligament of the knee. Arthroscopy: The Journal of Arthroscopic & Related Surgery. 2014 Nov 1;30(11):1475-82.
-
Question 19 of 55
19. Question
A 61 year old female presents to your office with complaints of a firm mass that has formed in the palm of her hand over the last month. She has noticed that she can’t fully extend her ring finger quite as much as her other fingers over the last month as well. On physical exam the patient has a palpable nodule on the palmar side of the 4th metacarpal that is non-painful to palpation (figure 1). You suspect the patient has early Dupuytren’s disease. Approximately what percentage of patients will progress from early Dupuytren’s to having a flexion deformity of the digit?
Correct
Dupuytren’s disease starts has a painless nodule in the pretendinous bands of the palmar fascia. The disease can progress as the involved finger, or fingers, start to develop a flexion deformity at the MCP and PIP joints and a thickened palmar cord. The flexion deformity increases as the palmar cord thickens and shortens. Observation is recommended until the involved finger has a flexion deformity at the MCP joint of 30 degrees or 15 degrees at the PIP joint (generally the stage of hand impairment). Not all patients with early stage disease will develop flexion deformities. Only 20% of patients with early disease will develop a flexion deformity in 7 years and about 35% in 18 years. Patients that have a family history and bilateral involvement are more likely to progress. 1,2
Answer B.
References
- Gil, Joseph A. MD; Akelman, Matthew R. MD; Hresko, Andrew M. MD; Akelman, Edward MD. Current Concepts in the Management of Dupuytren Disease of the Hand. Journal of the American Academy of Orthopaedic Surgeons 29(11):p 462-469, June 1, 2021. | DOI: 10.5435/JAAOS-D-20-00190
- Rayan, Ghazi M. Dupuytren Disease: Anatomy, Pathology, Presentation, and Treatment. J Bone Joint Surg Am, 89(1):189-198 | Selected Instructional Course Lecture | January, 2007
Incorrect
Dupuytren’s disease starts has a painless nodule in the pretendinous bands of the palmar fascia. The disease can progress as the involved finger, or fingers, start to develop a flexion deformity at the MCP and PIP joints and a thickened palmar cord. The flexion deformity increases as the palmar cord thickens and shortens. Observation is recommended until the involved finger has a flexion deformity at the MCP joint of 30 degrees or 15 degrees at the PIP joint (generally the stage of hand impairment). Not all patients with early stage disease will develop flexion deformities. Only 20% of patients with early disease will develop a flexion deformity in 7 years and about 35% in 18 years. Patients that have a family history and bilateral involvement are more likely to progress. 1,2
Answer B.
References
- Gil, Joseph A. MD; Akelman, Matthew R. MD; Hresko, Andrew M. MD; Akelman, Edward MD. Current Concepts in the Management of Dupuytren Disease of the Hand. Journal of the American Academy of Orthopaedic Surgeons 29(11):p 462-469, June 1, 2021. | DOI: 10.5435/JAAOS-D-20-00190
- Rayan, Ghazi M. Dupuytren Disease: Anatomy, Pathology, Presentation, and Treatment. J Bone Joint Surg Am, 89(1):189-198 | Selected Instructional Course Lecture | January, 2007
-
Question 20 of 55
20. Question
A 21 year old male presents to urgent care with a right elbow injury he sustained a few hours earlier. He fell on the right elbow after slipping on ice in his driveway. AP and lateral xrays show a minimally displaced olecranon fracture (figures 1 and 2). He is able to actively extend his elbow to -30 degrees of extension with moderate pain. He has pain to palpation over the tip of the olecranon. What is the best initial treatment?
Correct
The most common cause of an olecranon fracture is a direct blow to the elbow. Fracture patterns can range from a simple transverse (this patient’s pattern) to dislocated, comminuted, intra-articular fractures. The triceps attaches to the olecranon process and active elbow extension and passive elbow flexion beyond 90 degrees causes the triceps to place displacing forces on the proximal fracture fragment. Treatment of non-displaced or minimally displaced fractures (with an intact joint space and less than 2mm of displacement) includes immobilization with a long arm splint or cast with the elbow in 45 to 90 degrees of flexion. The elbow should be immobilized for 3 weeks in a long arm splint or cast and then light range of motion started. Patients shouldn’t flex the elbow beyond 90 degrees until there is radiographic signs of healing (to avoid tensioning the triceps and displacing the fracture). A range of motion brace can be used to restrict patients to < 90 degrees of flexion until 6 weeks post-injury or when bridging callus can be seen on xray. 1,2
Answer C.
References
- Veillette CJ, Steinmann SP. Olecranon fractures. Orthopedic Clinics of North America. 2008 Apr 1;39(2):229-36.
- Hak DJ, Golladay GJ. Olecranon fractures: treatment options. JAAOS-Journal of the American Academy of Orthopaedic Surgeons. 2000 Jul 1;8(4):266-75.
Incorrect
The most common cause of an olecranon fracture is a direct blow to the elbow. Fracture patterns can range from a simple transverse (this patient’s pattern) to dislocated, comminuted, intra-articular fractures. The triceps attaches to the olecranon process and active elbow extension and passive elbow flexion beyond 90 degrees causes the triceps to place displacing forces on the proximal fracture fragment. Treatment of non-displaced or minimally displaced fractures (with an intact joint space and less than 2mm of displacement) includes immobilization with a long arm splint or cast with the elbow in 45 to 90 degrees of flexion. The elbow should be immobilized for 3 weeks in a long arm splint or cast and then light range of motion started. Patients shouldn’t flex the elbow beyond 90 degrees until there is radiographic signs of healing (to avoid tensioning the triceps and displacing the fracture). A range of motion brace can be used to restrict patients to < 90 degrees of flexion until 6 weeks post-injury or when bridging callus can be seen on xray. 1,2
Answer C.
References
- Veillette CJ, Steinmann SP. Olecranon fractures. Orthopedic Clinics of North America. 2008 Apr 1;39(2):229-36.
- Hak DJ, Golladay GJ. Olecranon fractures: treatment options. JAAOS-Journal of the American Academy of Orthopaedic Surgeons. 2000 Jul 1;8(4):266-75.
-
Question 21 of 55
21. Question
A 12 year old girl presents to urgent care with left lateral foot pain for the past few weeks. She is in soccer season right now and the pain is made worse when she runs. She denies a history of a foot injury in the past. On physical exam she has tenderness to palpation at the proximal 5th metatarsal. There is no surrounding bruising or swelling and she can move the foot without pain. The white arrow on the oblique xray (figure 1) points to where the patient is having pain. The radiology reading on the xray is a possible fracture of the proximal 5th metatarsal. What is the next best step in treatment?
Correct
The patient is presenting with lateral foot pain which is likely due to traction apophysitis and a painful ossicle, or from a combination of them both. In the absence of significant swelling and ecchymosis, a fracture is unlikely. Given the patients age and the presence of open growth plates, traction apophysitis of the 5th metatarsal or Iselin’s Disease is a likely component causing the patients pain. The peroneus brevis tendon attaches to the proximal 5th metatarsal and can cause a traction apophysitis with frequent ankle motion. High impact activities like gymnastics and soccer are more prone to symptoms and the condition commonly affects kids between 9 and 13 years of age. The apophysis generally closes by ages 14-16 as the condition resolves. Treatment of Iselin’s disease includes rest, ice, and activity modification. The patient also appears to have a large ossicle, or os vesalianum, which is often mistaken for an avulsion fracture. Os vesalianum is seen as a well-corticated rounded bone at the proximal 5th metatarsal. Ossicles are generally found incidentally on xray but can become painful as they inflame surrounding soft tissues. 1,2
Answer A.
References
- Murphy, Robert F. MD; Van Nortwick, Sara S. MD; Jones, Richard MD; Mooney, James F. III MD. Evaluation and Management of Common Accessory Ossicles of the Foot and Ankle in Children and Adolescents. Journal of the American Academy of Orthopaedic Surgeons 29(7):p e312-e321, April 1, 2021. | DOI: 10.5435/JAAOS-D-20-00218
- Deniz G, Kose O, Guneri B, Duygun F. Traction apophysitis of the fifth metatarsal base in a child: Iselin’s disease. BMJ Case Rep. 2014 May 15;2014:bcr2014204687. doi: 10.1136/bcr-2014-204687. PMID: 24832713; PMCID: PMC4025211.
Incorrect
The patient is presenting with lateral foot pain which is likely due to traction apophysitis and a painful ossicle, or from a combination of them both. In the absence of significant swelling and ecchymosis, a fracture is unlikely. Given the patients age and the presence of open growth plates, traction apophysitis of the 5th metatarsal or Iselin’s Disease is a likely component causing the patients pain. The peroneus brevis tendon attaches to the proximal 5th metatarsal and can cause a traction apophysitis with frequent ankle motion. High impact activities like gymnastics and soccer are more prone to symptoms and the condition commonly affects kids between 9 and 13 years of age. The apophysis generally closes by ages 14-16 as the condition resolves. Treatment of Iselin’s disease includes rest, ice, and activity modification. The patient also appears to have a large ossicle, or os vesalianum, which is often mistaken for an avulsion fracture. Os vesalianum is seen as a well-corticated rounded bone at the proximal 5th metatarsal. Ossicles are generally found incidentally on xray but can become painful as they inflame surrounding soft tissues. 1,2
Answer A.
References
- Murphy, Robert F. MD; Van Nortwick, Sara S. MD; Jones, Richard MD; Mooney, James F. III MD. Evaluation and Management of Common Accessory Ossicles of the Foot and Ankle in Children and Adolescents. Journal of the American Academy of Orthopaedic Surgeons 29(7):p e312-e321, April 1, 2021. | DOI: 10.5435/JAAOS-D-20-00218
- Deniz G, Kose O, Guneri B, Duygun F. Traction apophysitis of the fifth metatarsal base in a child: Iselin’s disease. BMJ Case Rep. 2014 May 15;2014:bcr2014204687. doi: 10.1136/bcr-2014-204687. PMID: 24832713; PMCID: PMC4025211.
-
Question 22 of 55
22. Question
A 15 year old male presents to your office with left sided hip pain after a track injury a few days ago. He was taking off to sprint when he felt a pop in the hip and had to immediately stop running. On physical exam he has pain over the anterior superior iliac spine and weakness with active hip flexion. AP xray of the pelvis is shown in figure 1. What is the most likely diagnosis?
Correct
An anterior–superior iliac spine (ASIS) avulsion fractures are uncommon injuries that occur in adolescence as the iliac crest is at its weakest during this period of skeletal growth. A sudden hip flexion or contraction of the sartorius or the tenscia fascia lata during the starting phase of running or jumping can avulse the apophysis. The diagnosis is generally made with clinically findings and xrays and advanced imaging is often not necessary. Treatment involves a 2-3 week period of rest and avoidance of hip flexion exercises. Crutches and partial weight bearing may be necessary initially. Fractures with over 3 cm of displacement may require surgery with an open reduction and internal fixation. The most common avulsion injury of the pelvis is the ischial tuberosity from the hamstrings, followed by the ASIS by the sartorius, then the anterior inferior iliac spine from the rectus femoris. 1,2
Answer A.
References
- Bendeddouche I, Jean-Luc BB, Poiraudeau S, Nys A. Anterior superior iliac spine avulsion in a young soccer player. Annals of physical and rehabilitation medicine. 2010 Nov 1;53(9):584-90.
- Kautzner J, Trc T, Havlas V. Comparison of conservative against surgical treatment of anterior–superior iliac spine avulsion fractures in children and adolescents. International orthopaedics. 2014 Jul;38:1495-8.
Incorrect
An anterior–superior iliac spine (ASIS) avulsion fractures are uncommon injuries that occur in adolescence as the iliac crest is at its weakest during this period of skeletal growth. A sudden hip flexion or contraction of the sartorius or the tenscia fascia lata during the starting phase of running or jumping can avulse the apophysis. The diagnosis is generally made with clinically findings and xrays and advanced imaging is often not necessary. Treatment involves a 2-3 week period of rest and avoidance of hip flexion exercises. Crutches and partial weight bearing may be necessary initially. Fractures with over 3 cm of displacement may require surgery with an open reduction and internal fixation. The most common avulsion injury of the pelvis is the ischial tuberosity from the hamstrings, followed by the ASIS by the sartorius, then the anterior inferior iliac spine from the rectus femoris. 1,2
Answer A.
References
- Bendeddouche I, Jean-Luc BB, Poiraudeau S, Nys A. Anterior superior iliac spine avulsion in a young soccer player. Annals of physical and rehabilitation medicine. 2010 Nov 1;53(9):584-90.
- Kautzner J, Trc T, Havlas V. Comparison of conservative against surgical treatment of anterior–superior iliac spine avulsion fractures in children and adolescents. International orthopaedics. 2014 Jul;38:1495-8.
-
Question 23 of 55
23. Question
A 6 year old girl presents with her mother to your office with right elbow pain after she tried to do a cartwheel and fell awkwardly earlier in the day. On physical exam she has pain to palpation over the medial elbow without a deformity or bruising. There is no significant valgus instability to the elbow. AP xrays of the right elbow and left elbow side by side (figure 1) shows increased (2-3 mm) displacement of the medial condyle apophysis on the right. What is the best treatment of this minimally displaced medial epicondyle fracture?
Correct
The medial condyle is the last apophysis to fuse in the distal humerus, generally between 15 and 20 years of age, and is commonly injury in adolescence. The medial condyle is an attachment site for the flexor-pronator mass and the ulnar collateral ligament. The mechanism of injury is generally a sudden valgus force to the elbow during a fall on an outstretched hand with the elbow in extension. It can often be difficult to distinguish the fracture so contralateral films for comparison can be helpful. The amount of displacement that can be accepted before surgery is required is still debated. Most patients without valgus instability and minimally displaced fractures will do well with 1-3 weeks of immobilization followed by gentle range of motion exercises. Loss of elbow extension raises concern of a completely displaced condyle fracture with the fragment trapped in the elbow joint. Open reduction and internal fixation is the surgical approach of choice with significant displacement or elbow instability. 1,2
Answer B.
References
- Gottschalk, Hilton P. MD; Eisner, Eric MD; Hosalkar, Harish S. MD. Medial Epicondyle Fractures in the Pediatric Population. Journal of the American Academy of Orthopaedic Surgeons 20(4):p 223-232, April 2012. | DOI: 10.5435/JAAOS-20-04-223
- Hennrikus WL. Elbow disorders in the young athlete. Operative Techniques in Sports Medicine. 2006 Jul 1;14(3):165-72.
Incorrect
The medial condyle is the last apophysis to fuse in the distal humerus, generally between 15 and 20 years of age, and is commonly injury in adolescence. The medial condyle is an attachment site for the flexor-pronator mass and the ulnar collateral ligament. The mechanism of injury is generally a sudden valgus force to the elbow during a fall on an outstretched hand with the elbow in extension. It can often be difficult to distinguish the fracture so contralateral films for comparison can be helpful. The amount of displacement that can be accepted before surgery is required is still debated. Most patients without valgus instability and minimally displaced fractures will do well with 1-3 weeks of immobilization followed by gentle range of motion exercises. Loss of elbow extension raises concern of a completely displaced condyle fracture with the fragment trapped in the elbow joint. Open reduction and internal fixation is the surgical approach of choice with significant displacement or elbow instability. 1,2
Answer B.
References
- Gottschalk, Hilton P. MD; Eisner, Eric MD; Hosalkar, Harish S. MD. Medial Epicondyle Fractures in the Pediatric Population. Journal of the American Academy of Orthopaedic Surgeons 20(4):p 223-232, April 2012. | DOI: 10.5435/JAAOS-20-04-223
- Hennrikus WL. Elbow disorders in the young athlete. Operative Techniques in Sports Medicine. 2006 Jul 1;14(3):165-72.
-
Question 24 of 55
24. Question
A 45 year old male presents to your office with bilateral hand numbness for at least 2 months. The numbness seems to affect all the fingers and he hasn’t recognized a cause. He had a recent EMG study which showed possible ulnar neuropathy at the wrist, but the results were unclear. Over the last week he has also noticed the tip of his left small finger turn purple (figure 1 and 2) and the finger tip is starting to get extremely painful. He notes to smoke one pack per day but is otherwise healthy. What is the next best step in treatment?
Correct
Raynaud’s disease is characterized by vasospasm of the digital arteries that results in the fingers turning cold, numb, white, and in severe cases, purple. The vasospasm is usually a result of cold temperatures but the etiology of the disease is unknown. Smoking is associated with a higher prevalence of Raynaud’s disease as nicotine enhances vasoconstriction. First line treatment is avoidance of cold temperatures (wearing gloves) and smoking cessation. Calcium channel blockers and topical nitrates are also a common first line treatment to help dilate the digital capillaries. Raynaud’s syndrome or phenomenon is a form of the disease with an identifiable cause such as connective tissue disorders systemic lupus erythematosis and scleroderma, drug induced (beta-blockers, anti-migraine), obstructive disease (atherosclerosis and emboli), and work related (vibration, cold injury working with frozen foods) . 1,2
Answer A.
References
- Temprano KK. A Review of Raynaud’s Disease. Mo Med. 2016 Mar-Apr;113(2):123-6. PMID: 27311222; PMCID: PMC6139949.
- Turton EP, Kent PJ, Kester RC. VASCULAR REVIEW: The aetiology of Raynaud’s phenomenon. Cardiovascular Surgery. 1998 Oct 1;6(5):431-40.
Incorrect
Raynaud’s disease is characterized by vasospasm of the digital arteries that results in the fingers turning cold, numb, white, and in severe cases, purple. The vasospasm is usually a result of cold temperatures but the etiology of the disease is unknown. Smoking is associated with a higher prevalence of Raynaud’s disease as nicotine enhances vasoconstriction. First line treatment is avoidance of cold temperatures (wearing gloves) and smoking cessation. Calcium channel blockers and topical nitrates are also a common first line treatment to help dilate the digital capillaries. Raynaud’s syndrome or phenomenon is a form of the disease with an identifiable cause such as connective tissue disorders systemic lupus erythematosis and scleroderma, drug induced (beta-blockers, anti-migraine), obstructive disease (atherosclerosis and emboli), and work related (vibration, cold injury working with frozen foods) . 1,2
Answer A.
References
- Temprano KK. A Review of Raynaud’s Disease. Mo Med. 2016 Mar-Apr;113(2):123-6. PMID: 27311222; PMCID: PMC6139949.
- Turton EP, Kent PJ, Kester RC. VASCULAR REVIEW: The aetiology of Raynaud’s phenomenon. Cardiovascular Surgery. 1998 Oct 1;6(5):431-40.
-
Question 25 of 55
25. Question
A 26 year old delivery driver presents to your office after being bitten by a dog on his left ring finger earlier in the day. The dog clenched down on the tip of his finger and he now has an abrasion and subungual hematoma to the ring finger but the nail and skin are intact. AP xray of the distal phalanx of the ring finger shows a transverse distal phalanx fracture (figure 1). What is the best prophylactic antibiotic for this injury?
Correct
Dog bites are the most common animal bites and the dog is known by the victim in most cases. Dog bites are often poly microbial with common aerobic bacteria such as Pasteurella, Streptococcus, Staphylococcus, Neisseria, and Corynebacterium, among others. Anerobic bacteria can include Fusobacterium, Porphyromonas, Propionibacterium, and Bacteroides. For dog bite wounds mixed aerobes and anaerobes constitute >50% of wound cultures with an average of 5 different bacterial isolates. Pasteurella was the most common isolate found in dog wounds with 50% which is under the 75% found in cat bite wounds. Dog bite wounds of the hand should be treated empirically with amoxicillin-clavulanate which is the oral antibiotic of choice for human, dog, and cat bites. If patients have persistent pain beyond 2-3 weeks osteomyelitis (although rare after dog and cat bites) should be ruled out with an MRI. 1,2
Answer C.
References
- Kennedy, Stephen A. MD; Stoll, Laura E. MD; Lauder, Alexander S. MD. Human and Other Mammalian Bite Injuries of the Hand: Evaluation and Management. Journal of the American Academy of Orthopaedic Surgeons 23(1):p 47-57, January 2015. | DOI: 10.5435/JAAOS-23-01-47
- Lee R, Lee HY, Kim JH, Han YS, Kim DC, Tark KC. Acute osteomyelitis in the hand due to dog bite injury: a report of 3 cases. Archives of plastic surgery. 2017 Sep;44(05):444-8.
Incorrect
Dog bites are the most common animal bites and the dog is known by the victim in most cases. Dog bites are often poly microbial with common aerobic bacteria such as Pasteurella, Streptococcus, Staphylococcus, Neisseria, and Corynebacterium, among others. Anerobic bacteria can include Fusobacterium, Porphyromonas, Propionibacterium, and Bacteroides. For dog bite wounds mixed aerobes and anaerobes constitute >50% of wound cultures with an average of 5 different bacterial isolates. Pasteurella was the most common isolate found in dog wounds with 50% which is under the 75% found in cat bite wounds. Dog bite wounds of the hand should be treated empirically with amoxicillin-clavulanate which is the oral antibiotic of choice for human, dog, and cat bites. If patients have persistent pain beyond 2-3 weeks osteomyelitis (although rare after dog and cat bites) should be ruled out with an MRI. 1,2
Answer C.
References
- Kennedy, Stephen A. MD; Stoll, Laura E. MD; Lauder, Alexander S. MD. Human and Other Mammalian Bite Injuries of the Hand: Evaluation and Management. Journal of the American Academy of Orthopaedic Surgeons 23(1):p 47-57, January 2015. | DOI: 10.5435/JAAOS-23-01-47
- Lee R, Lee HY, Kim JH, Han YS, Kim DC, Tark KC. Acute osteomyelitis in the hand due to dog bite injury: a report of 3 cases. Archives of plastic surgery. 2017 Sep;44(05):444-8.
-
Question 26 of 55
26. Question
A 34 year old right handed dominant female fell off a bike two weeks ago and landed on her outstretched right hand. She had some pain, swelling and bruising initially. She went to an urgent care where xrays were performed and read as negative (figure 1). On physical exam of the right wrist she has some bruising extending over the radial aspect of the wrist extending up to the thumb metacarpal. She has slightly diminished wrist motion with some discomfort. She is locally tender over the snuff box and scaphoid tuberosity. An MRI was performed of the wrist showing a non-displaced scaphoid waist fracture. Which statement below is true regarding advanced imaging for diagnosing occult scaphoid fractures?
Correct
An occult scaphoid fracture is often suspected in the setting of a fall on an outstretched hand, anatomical snuffbox tenderness, and negative wrist xrays. Because scaphoid fractures are prone to non-union and degenerative changes if the fracture is missed, it is critical to establish a timely diagnosis. At the same time you don’t want to overtreat patients and unnecessarily have them miss work or enjoyable activities. In general these patients can be treated with wrist immobilization and re-examined in two weeks. When an occult scaphoid fracture is suspected thereafter the question arises if an MRI should be performed or if patients should be empirically treated with cast immobilization for 6 weeks. Karl et al found that ordering an MRI was associated with lower cost and improved outcomes compared to empiric cast treatment. MRI has the added benefit of diagnosing soft tissue pathology. If an occult fracture is found on MRI a short arm cast for 6 weeks is the recommended treatment (thumb spica casting has no benefit over a short arm cast for the treatment of scaphoid fractures. 1,2
Answer B.
References
- Karl, John W. MD, MPH1; Swart, Eric MD1; Strauch, Robert J. MD1. Diagnosis of Occult Scaphoid Fractures: A Cost-Effectiveness Analysis. The Journal of Bone and Joint Surgery 97(22):p 1860-1868, November 18, 2015. | DOI: 10.2106/JBJS.O.00099
- Duckworth AD, Ring D, McQueen MM. Assessment of the suspected fracture of the scaphoid. The Journal of Bone & Joint Surgery British Volume. 2011 Jun 1;93(6):713-9.
Incorrect
An occult scaphoid fracture is often suspected in the setting of a fall on an outstretched hand, anatomical snuffbox tenderness, and negative wrist xrays. Because scaphoid fractures are prone to non-union and degenerative changes if the fracture is missed, it is critical to establish a timely diagnosis. At the same time you don’t want to overtreat patients and unnecessarily have them miss work or enjoyable activities. In general these patients can be treated with wrist immobilization and re-examined in two weeks. When an occult scaphoid fracture is suspected thereafter the question arises if an MRI should be performed or if patients should be empirically treated with cast immobilization for 6 weeks. Karl et al found that ordering an MRI was associated with lower cost and improved outcomes compared to empiric cast treatment. MRI has the added benefit of diagnosing soft tissue pathology. If an occult fracture is found on MRI a short arm cast for 6 weeks is the recommended treatment (thumb spica casting has no benefit over a short arm cast for the treatment of scaphoid fractures. 1,2
Answer B.
References
- Karl, John W. MD, MPH1; Swart, Eric MD1; Strauch, Robert J. MD1. Diagnosis of Occult Scaphoid Fractures: A Cost-Effectiveness Analysis. The Journal of Bone and Joint Surgery 97(22):p 1860-1868, November 18, 2015. | DOI: 10.2106/JBJS.O.00099
- Duckworth AD, Ring D, McQueen MM. Assessment of the suspected fracture of the scaphoid. The Journal of Bone & Joint Surgery British Volume. 2011 Jun 1;93(6):713-9.
-
Question 27 of 55
27. Question
A 66 year old female presents to your office with right hip pain for the past year. She has tried courses of oral anti-inflammatories and rest but the pain keeps returning. She is looking forward to a European vacation in 6 months and would like to know the most predictable treatment option for pain relief. On physical exam she has almost no internal rotation and painful passive hip motion. AP pelvis xray is shown in figure 1. Which treatment option would give this patient the most predictable pain relief for her vacation?
Correct
The most predictable treatment option for this patient is a non-cemented total hip replacement. This patient has severe osteoarthritis with subchondral sclerosis, large osteophytes, and limited hip motion. Physical therapy is a good option for patients with mild or moderate osteoarthritis but unlikely to provide any benefit in patients with severe osteoarthritis. An intra-articular injection provides short term relief (generally 2-3 months) but has been shown to cause an increased risk of an infection if performed within 3 months of hip arthroplasty. So if the injection failed to relieve her pain she would unlikely be able get her hip replaced before her vacation (arthroplasty must be performed >3 months after an intra-articular steroid injection). Hip arthroscopy is not recommended in patients of advanced age (>60) and with advanced arthritis as these patients are the most likely to do poorly and require a conversion to a total hip arthroplasty. In general, cemented total hip replacements are only used for patients over the age of 75 years as the benefit of cement is a lower risk of periprosthetic fracture in this age group 1.2
Answer D.
References
1. Werner BC, Cancienne JM, Browne JA. The Timing of Total Hip Arthroplasty After Intraarticular Hip Injection Affects Postoperative Infection Risk. J Arthroplasty. 2016 Apr;31(4):820-3.
2. Hannon, Charles P. MD, MBA; Delanois, Ronald E. MD; Nandi, Sumon MD, MBA; Fillingham, Yale MD; Management of Osteoarthritis of the Hip Work Group Staff of the American Academy of Orthopaedic Surgeons; Management of Osteoarthritis of the Hip Work Group Staff of the American Academy of Orthopaedic Surgeons. American Academy of Orthopaedic Surgeons Clinical Practice Guideline Summary Management of Osteoarthritis of the Hip. Journal of the American Academy of Orthopaedic Surgeons ():10.5435/JAAOS-D-24-00420, June 6, 2024. |Incorrect
The most predictable treatment option for this patient is a non-cemented total hip replacement. This patient has severe osteoarthritis with subchondral sclerosis, large osteophytes, and limited hip motion. Physical therapy is a good option for patients with mild or moderate osteoarthritis but unlikely to provide any benefit in patients with severe osteoarthritis. An intra-articular injection provides short term relief (generally 2-3 months) but has been shown to cause an increased risk of an infection if performed within 3 months of hip arthroplasty. So if the injection failed to relieve her pain she would unlikely be able get her hip replaced before her vacation (arthroplasty must be performed >3 months after an intra-articular steroid injection). Hip arthroscopy is not recommended in patients of advanced age (>60) and with advanced arthritis as these patients are the most likely to do poorly and require a conversion to a total hip arthroplasty. In general, cemented total hip replacements are only used for patients over the age of 75 years as the benefit of cement is a lower risk of periprosthetic fracture in this age group 1.2
Answer D.
References
1. Werner BC, Cancienne JM, Browne JA. The Timing of Total Hip Arthroplasty After Intraarticular Hip Injection Affects Postoperative Infection Risk. J Arthroplasty. 2016 Apr;31(4):820-3.
2. Hannon, Charles P. MD, MBA; Delanois, Ronald E. MD; Nandi, Sumon MD, MBA; Fillingham, Yale MD; Management of Osteoarthritis of the Hip Work Group Staff of the American Academy of Orthopaedic Surgeons; Management of Osteoarthritis of the Hip Work Group Staff of the American Academy of Orthopaedic Surgeons. American Academy of Orthopaedic Surgeons Clinical Practice Guideline Summary Management of Osteoarthritis of the Hip. Journal of the American Academy of Orthopaedic Surgeons ():10.5435/JAAOS-D-24-00420, June 6, 2024. | -
Question 28 of 55
28. Question
A 56 year old male presents to your office with pain and swelling at the distal interphalangeal (DIP) joint of the long finger for the past 2 weeks. He works in landscaping and he injured the finger 6 years ago when a large rock fell on it. He did not seek treatment at the time and the finger has been intermittently swollen since, but worse over the last couple of weeks. He admits that the joint has been larger over the last few years but the redness at the joint is new. On physical exam, he has a large area of swelling at the DIP joint that is slightly erythematous. The area is firm and tender, and without fluctuance, warmth or signs of infection (figures 1 and 2). X-rays show there to be severe osteoarthritis at the DIP joint of third finger (figures 3 and 4). What is the best treatment option?
Correct
Heberden’s nodes are characterized by enlargement of the distal interphalangeal joint (DIP) and are associated with osteoarthritis. This patient likely had post-traumatic osteoarthritis develop after his previously injury 6 years ago, as is the case in many manual laborers. The differential diagnosis includes a ganglion cyst and gout, although these are usually associated with fluctuance on physical exam consistent with a fluid collection. Gout characteristically affects the larger MCP and PIP joints and is often associated with subcutaneous tophi. Osteophytes on x-ray are not always present with Heberden’s nodes, but in the case of this patient, his large osteophytes are most likely contributing to the enlargement of the DIP joint. Joint inflammation associated with arthritis can present with slight warmth and erythema which can mimic an infection. The clinical picture, x-ray findings, and exam are critical components of establishing a diagnosis. 1,2
Given the patients history and radiographic findings, the most likely diagnosis is an arthritic flare-up of his DIP joint which is best treated with a splint for 1-2 weeks along with an oral steroid.
Answer C.
References
1. Urbano FL. Heberden’s nodes. Hospital Physician. 2001 Jul;38:29-31.
2. Zu Reckendorf GM, Dahmam A. Hand involvement in gout. Hand Surgery and Rehabilitation. 2018 Jul 1;37(4):197-201Incorrect
Heberden’s nodes are characterized by enlargement of the distal interphalangeal joint (DIP) and are associated with osteoarthritis. This patient likely had post-traumatic osteoarthritis develop after his previously injury 6 years ago, as is the case in many manual laborers. The differential diagnosis includes a ganglion cyst and gout, although these are usually associated with fluctuance on physical exam consistent with a fluid collection. Gout characteristically affects the larger MCP and PIP joints and is often associated with subcutaneous tophi. Osteophytes on x-ray are not always present with Heberden’s nodes, but in the case of this patient, his large osteophytes are most likely contributing to the enlargement of the DIP joint. Joint inflammation associated with arthritis can present with slight warmth and erythema which can mimic an infection. The clinical picture, x-ray findings, and exam are critical components of establishing a diagnosis. 1,2
Given the patients history and radiographic findings, the most likely diagnosis is an arthritic flare-up of his DIP joint which is best treated with a splint for 1-2 weeks along with an oral steroid.
Answer C.
References
1. Urbano FL. Heberden’s nodes. Hospital Physician. 2001 Jul;38:29-31.
2. Zu Reckendorf GM, Dahmam A. Hand involvement in gout. Hand Surgery and Rehabilitation. 2018 Jul 1;37(4):197-201 -
Question 29 of 55
29. Question
A 36 year old male presents with left foot pain after a fall in his yard a few hours ago. He was running and slid on wet grass as his foot twisted awkwardly. He had immediate pain and was unable to bear weight. On physical exam he has pain to palpation and swelling to the forefoot without an obvious deformity. AP and lateral x-rays of the foot are shown in figures 1 and 2. CT of the left foot shows comminuted displaced fractures of the base of the second, third, and fourth metatarsals with intra-articular extension. There is lateral displacement of the metatarsal shafts relative to the proximal fracture fragments. What is the best treatment option?
Correct
The patient sustained a Lisfranc fracture dislocation with lateral displacement of the second, third, and fourth metatarsals. The Lisfranc joint consists of the tarsometatarsal joints which are reinforced by the Lisfranc ligament complex, intermetatarsal ligaments, and the TMT joint capsules. Injuries to the Lisfranc joint can range from subtle ligament sprains to fracture dislocations. Any change of normal anatomy on radiographs represents an unstable injury that warrants surgical fixation. Subtle or minimal displacement of the Lisfranc joint can be treated with closed reduction and percutaneous fixation with screws or pins. Fractures with displacement and comminution are best treated with ORIF with bridge plating for its superior reduction quality and extra-articular fixation. Bridge plating helps preserve metatarsal length and avoids placing intra-articular screws. Functional outcomes can still be fair to poor despite adequate fixation with ORIF due to a high rate of non-union and post-traumatic arthritis. A midfoot fusion is the definitive treatment option should ORIF fail. 1,2
Answer C.
References
1. Joseph, Noah M. MD, MS; Patel, Raahil MD; Freedman, Clara BS; Cox, Kaitlyn BS; Mir, Hassan R. MD, MBA. Open Reduction And Internal Fixation of Tarsometatarsal (Lisfranc) Fracture Dislocations—Is Arthrodesis Necessary?. Journal of the American Academy of Orthopaedic Surgeons 32(4):p 178-185, February 15, 2024. |
2. Benirschke SK, Meinberg E, Anderson SA, Jones CB, Cole PA. Fractures and dislocations of the midfoot: Lisfranc and Chopart injuries. J Bone Joint Surg Am, 2012 Jul 18; 94(14); 1325-37Incorrect
The patient sustained a Lisfranc fracture dislocation with lateral displacement of the second, third, and fourth metatarsals. The Lisfranc joint consists of the tarsometatarsal joints which are reinforced by the Lisfranc ligament complex, intermetatarsal ligaments, and the TMT joint capsules. Injuries to the Lisfranc joint can range from subtle ligament sprains to fracture dislocations. Any change of normal anatomy on radiographs represents an unstable injury that warrants surgical fixation. Subtle or minimal displacement of the Lisfranc joint can be treated with closed reduction and percutaneous fixation with screws or pins. Fractures with displacement and comminution are best treated with ORIF with bridge plating for its superior reduction quality and extra-articular fixation. Bridge plating helps preserve metatarsal length and avoids placing intra-articular screws. Functional outcomes can still be fair to poor despite adequate fixation with ORIF due to a high rate of non-union and post-traumatic arthritis. A midfoot fusion is the definitive treatment option should ORIF fail. 1,2
Answer C.
References
1. Joseph, Noah M. MD, MS; Patel, Raahil MD; Freedman, Clara BS; Cox, Kaitlyn BS; Mir, Hassan R. MD, MBA. Open Reduction And Internal Fixation of Tarsometatarsal (Lisfranc) Fracture Dislocations—Is Arthrodesis Necessary?. Journal of the American Academy of Orthopaedic Surgeons 32(4):p 178-185, February 15, 2024. |
2. Benirschke SK, Meinberg E, Anderson SA, Jones CB, Cole PA. Fractures and dislocations of the midfoot: Lisfranc and Chopart injuries. J Bone Joint Surg Am, 2012 Jul 18; 94(14); 1325-37 -
Question 30 of 55
30. Question
A 68 year old female presents with left wrist pain after a fall 2 days ago. She is also complaining of numbness to the thumb and index finger since the fall. On physical exam she has a mild deformity to the left wrist with pain to palpation over the distal radius. She is able to move her fingers but has a slight decrease in sensation in a medial nerve distribution. AP and lateral xrays of the left wrist are shown in figures 1 and 2. What is the best treatment option?
Correct
Acute carpel tunnel syndrome (ACTS) can occur with displaced distal radius fractures. The incidence ranges from 4-17% of all distal radius fractures. ACTS with a distal radius fracture is thought to be caused by elevated compartment pressures in the carpel tunnel due to fracture displacement and hematoma formation. Displaced intra-articular fractures have the highest prevalence of ACTS. Symptoms include increasing pain and altered sensation in the median nerve distribution after a fracture. Symptoms can start within hours to days after the injury. Distinguishing between ACTS that requires carpel tunnel release vs. ACTS that will resolve over time is difficult to determine, however progressively worsening symptoms may be the best differentiator. Symptoms usually resolve with splinting, rest, ice and elevation over a period of 3-5 days. Patients presenting with ACTS with a displaced fracture should be closed reduced in a timely manner which can help resolve symptoms. If symptoms fail to resolve in 5-7 days than a carpel tunnel release can be performed at the time of ORIF of the wrist for displaced unstable fractures. 1,2
Answer B.
References
1. Leow JM, Clement ND, McQueen MM, Duckworth AD. The rate and associated risk factors for acute carpal tunnel syndrome complicating a fracture of the distal radius. Eur J Orthop Surg Traumatol. 2021 Jul;31(5):981-987.
2. Samuel, Thomas D. MBBS*; Jeffrey, Hamish MBBS*; Hayter, Edward MRCS*; Lee, George MRCS*; Little, Maximillian MRCS*; Hardman, John FRCS (Tr&Orth)*; Anakwe, Raymond E. FRCS Ed(Tr&Orth) Acute Carpal Tunnel Syndrome: Early Nerve Decompression and Surgical Stabilization for Bony Wrist Trauma. Plastic & Reconstructive Surgery-Global Open 11(4):p e4929, April 2023. |Incorrect
Acute carpel tunnel syndrome (ACTS) can occur with displaced distal radius fractures. The incidence ranges from 4-17% of all distal radius fractures. ACTS with a distal radius fracture is thought to be caused by elevated compartment pressures in the carpel tunnel due to fracture displacement and hematoma formation. Displaced intra-articular fractures have the highest prevalence of ACTS. Symptoms include increasing pain and altered sensation in the median nerve distribution after a fracture. Symptoms can start within hours to days after the injury. Distinguishing between ACTS that requires carpel tunnel release vs. ACTS that will resolve over time is difficult to determine, however progressively worsening symptoms may be the best differentiator. Symptoms usually resolve with splinting, rest, ice and elevation over a period of 3-5 days. Patients presenting with ACTS with a displaced fracture should be closed reduced in a timely manner which can help resolve symptoms. If symptoms fail to resolve in 5-7 days than a carpel tunnel release can be performed at the time of ORIF of the wrist for displaced unstable fractures. 1,2
Answer B.
References
1. Leow JM, Clement ND, McQueen MM, Duckworth AD. The rate and associated risk factors for acute carpal tunnel syndrome complicating a fracture of the distal radius. Eur J Orthop Surg Traumatol. 2021 Jul;31(5):981-987.
2. Samuel, Thomas D. MBBS*; Jeffrey, Hamish MBBS*; Hayter, Edward MRCS*; Lee, George MRCS*; Little, Maximillian MRCS*; Hardman, John FRCS (Tr&Orth)*; Anakwe, Raymond E. FRCS Ed(Tr&Orth) Acute Carpal Tunnel Syndrome: Early Nerve Decompression and Surgical Stabilization for Bony Wrist Trauma. Plastic & Reconstructive Surgery-Global Open 11(4):p e4929, April 2023. | -
Question 31 of 55
31. Question
A 26 year old male presents with left shoulder pain after diving off a cliff from a significant height of over 100 feet. After the dive he felt significant discomfort along the chest wall and developed some bruising in the axillary region as well as along the medial aspect of the upper arm on the left side. He has a mild deformity of the pectoralis major muscle which seems to be pronounced when he holds his arm up to his side (abduction and external rotation). Axillary view MRI of the left pectoralis region shows a torn and retracted pectoralis major tendon with a fluid filled gap (Figure 1). Without a surgical repair, the patient would have weakness with what motion?
Correct
The pectoralis major (PM) muscle originates from the sternum and adjacent first through sixth ribs and inserts on the proximal humerus just lateral to the bicipital groove. The clavicular head of the pectoralis major helps the deltoid forward flex the arm and the sternal head (the largest portion accounting for 80% of the muscle belly) internally rotates and adducts the humerus. The bench press exercise isolates the PM and is the most common mechanism of injury (particularly with downward deceleration). PM rupture occurs almost exclusively in males between the ages of 20 and 40. An acute injury is usually felt as a pop and sharp pain with subsequent bruising in the anterior chest and axilla. A deformity to the PM can best be seen with the shoulders in 90 degrees of abduction and external rotation. MRI is the study of choice to confirm the diagnosis of a PM tear. Treatment of muscle belly tears includes a sling and a 4-6 week period of rest. Open repair is indicated for tendon avulsion injuries and musculotendon tears. Non-operative treatment would lead to weakness with adduction and internal rotation. 1,2
Answer D.
References
- Kowalczuk, Marcin MD, FRCSC; Elmaraghy, Amr MD, FRCSC. Pectoralis Major Rupture: Evaluation and Management. Journal of the American Academy of Orthopaedic Surgeons 30(7):p e617-e627, April 1, 2022. | DOI: 10.5435/JAAOS-D-21-00541
- Provencher CM, Handfield K, Boniquit NT, Reiff SN, Sekiya JK, Romeo AA. Injuries to the pectoralis major muscle: diagnosis and management. The American Journal of Sports Medicine. 2010 Aug;38(8):1693-705.
Incorrect
The pectoralis major (PM) muscle originates from the sternum and adjacent first through sixth ribs and inserts on the proximal humerus just lateral to the bicipital groove. The clavicular head of the pectoralis major helps the deltoid forward flex the arm and the sternal head (the largest portion accounting for 80% of the muscle belly) internally rotates and adducts the humerus. The bench press exercise isolates the PM and is the most common mechanism of injury (particularly with downward deceleration). PM rupture occurs almost exclusively in males between the ages of 20 and 40. An acute injury is usually felt as a pop and sharp pain with subsequent bruising in the anterior chest and axilla. A deformity to the PM can best be seen with the shoulders in 90 degrees of abduction and external rotation. MRI is the study of choice to confirm the diagnosis of a PM tear. Treatment of muscle belly tears includes a sling and a 4-6 week period of rest. Open repair is indicated for tendon avulsion injuries and musculotendon tears. Non-operative treatment would lead to weakness with adduction and internal rotation. 1,2
Answer D.
References
- Kowalczuk, Marcin MD, FRCSC; Elmaraghy, Amr MD, FRCSC. Pectoralis Major Rupture: Evaluation and Management. Journal of the American Academy of Orthopaedic Surgeons 30(7):p e617-e627, April 1, 2022. | DOI: 10.5435/JAAOS-D-21-00541
- Provencher CM, Handfield K, Boniquit NT, Reiff SN, Sekiya JK, Romeo AA. Injuries to the pectoralis major muscle: diagnosis and management. The American Journal of Sports Medicine. 2010 Aug;38(8):1693-705.
-
Question 32 of 55
32. Question
A 15 year old female presents to your office with left knee pain after slipping on a wet surface as her knee buckled. She had immediate pain and later developed significant swelling that limited her mobility. On physical exam of the left knee she has no laxity with varus or valgus stress and she has a negative Lachman’s. She has moderate pain to palpation over the lateral femoral condyle and a large joint effusion. Her initial xrays were negative and MRI images of the left knee (figures 1 and 2) show bone marrow edema in the lateral femoral condyle. What is the best treatment option for this patient?
Correct
Bone marrow edema is a commonly found MRI finding with a wide differential from a traumatic cause to osteoarthritis, infection, and avascular necrosis, among others. Traumatic related bone marrow edema or a “bone bruise” on MRI is a common finding after knee injuries and is often associated with ACL and meniscal tears, patella dislocations, and collateral ligament injuries. Bone edema can be identified on the MRI as an area of altered signal of the bone with a high signal intensity on fat-suppressed, T2 weighted images. The pathogenesis of bone marrow edema is unclear but traumatic edema likely results from a fluid build up in the bone marrow. Why bone marrow edema causes pain is not entirely clear, however the theory is that increased interosseus pressure is the most likely cause. Knee pain associated with edema is generally made worse with weight bearing and high impact activities. Bone marrow edema is self-limited, and most cases will resolve within 2–4 months following conservative treatment. The most common treatment involves 3-6 weeks of partial weight bearing and avoidance of high impact activities. Patients can slowly begin high impact activities after pain free weight bearing, with the understanding it may take 4 months or longer to return to high impact activities pain free. 1,2
Answer B.
References
- Akhavan, Sam MD; Martinkovich, Stephen C. MD; Kasik, Connor DO; DeMeo, Patrick J. MD. Bone Marrow Edema, Clinical Significance, and Treatment Options: A Review. Journal of the American Academy of Orthopaedic Surgeons 28(20):p e888-e899, October 15, 2020. | DOI: 10.5435/JAAOS-D-20-00142
- Tarantino U, Greggi C, Cariati I, Caldora P, Capanna R, Capone A, Civinini R, Colagrande S, De Biase P, Falez F, Iolascon G, Maraghelli D, Masi L, Cerinic MM, Sessa G, Brandi ML. Bone Marrow Edema: Overview of Etiology and Treatment Strategies. J Bone Joint Surg Am. 2022 Jan 19;104(2):189-200. doi: 10.2106/JBJS.21.00300. PMID: 34780382.
Incorrect
Bone marrow edema is a commonly found MRI finding with a wide differential from a traumatic cause to osteoarthritis, infection, and avascular necrosis, among others. Traumatic related bone marrow edema or a “bone bruise” on MRI is a common finding after knee injuries and is often associated with ACL and meniscal tears, patella dislocations, and collateral ligament injuries. Bone edema can be identified on the MRI as an area of altered signal of the bone with a high signal intensity on fat-suppressed, T2 weighted images. The pathogenesis of bone marrow edema is unclear but traumatic edema likely results from a fluid build up in the bone marrow. Why bone marrow edema causes pain is not entirely clear, however the theory is that increased interosseus pressure is the most likely cause. Knee pain associated with edema is generally made worse with weight bearing and high impact activities. Bone marrow edema is self-limited, and most cases will resolve within 2–4 months following conservative treatment. The most common treatment involves 3-6 weeks of partial weight bearing and avoidance of high impact activities. Patients can slowly begin high impact activities after pain free weight bearing, with the understanding it may take 4 months or longer to return to high impact activities pain free. 1,2
Answer B.
References
- Akhavan, Sam MD; Martinkovich, Stephen C. MD; Kasik, Connor DO; DeMeo, Patrick J. MD. Bone Marrow Edema, Clinical Significance, and Treatment Options: A Review. Journal of the American Academy of Orthopaedic Surgeons 28(20):p e888-e899, October 15, 2020. | DOI: 10.5435/JAAOS-D-20-00142
- Tarantino U, Greggi C, Cariati I, Caldora P, Capanna R, Capone A, Civinini R, Colagrande S, De Biase P, Falez F, Iolascon G, Maraghelli D, Masi L, Cerinic MM, Sessa G, Brandi ML. Bone Marrow Edema: Overview of Etiology and Treatment Strategies. J Bone Joint Surg Am. 2022 Jan 19;104(2):189-200. doi: 10.2106/JBJS.21.00300. PMID: 34780382.
-
Question 33 of 55
33. Question
A 70 year old male presents to your office with left foot pain for 3 days after slipping on a rug and falling as he struck his lesser toes. He was initially seen at urgent care where xrays showed a dorsal dislocation of the second toe at the PIP joint (Figure 1). A closed reduction was attempted but was unsuccessful. A second attempt at closed reduction under fluoroscopy was also unsuccessful (Figure 2). He continues to have pain and swelling in the left second toe. At his point the second toe is unstable and dislocating. What is the best treatment option?
Correct
Lesser toe dislocations are uncommon injuries that can be complex to treat. Many traumatic interphalangeal joint dislocations are irreducible by closed means. An irreducible dislocation is defined as a dislocation that can’t be reduced without a surgical open reduction. Irreducible interphalangeal joint dislocations of the lesser toes can result from incarceration of the collateral ligaments, the plantar plate (most common), or the flexor tendon into the joint. Most commonly the dislocation is dorsal which results in rupture of the plantar plate with displacement and joint interposition. Open reduction, most commonly with a dorsal incision and arthrotomy, is required to remove the interposed tissue so the joint will reduce. The joint is then reduced and pinned in place with a k-wire to maintain the reduction. The k-wire pin is kept in place for 4-6 weeks as the joint stabilizes and the surrounding soft tissues heal. An interphalangeal joint fusion may be performed for complex fracture dislocations of the IP joint. 1,2
Answer C.
References
- Brunet JA, Tubin S. Traumatic dislocations of the lesser toes. Foot & ankle international. 1997 Jul;18(7):406-11.
- Córdoba-Fernández A. Management of nonreducible lesser toe interphalangeal dislocation: an unusual injury. Journal of the American Podiatric Medical Association. 2012 May 1;102(3):252-5.
Incorrect
Lesser toe dislocations are uncommon injuries that can be complex to treat. Many traumatic interphalangeal joint dislocations are irreducible by closed means. An irreducible dislocation is defined as a dislocation that can’t be reduced without a surgical open reduction. Irreducible interphalangeal joint dislocations of the lesser toes can result from incarceration of the collateral ligaments, the plantar plate (most common), or the flexor tendon into the joint. Most commonly the dislocation is dorsal which results in rupture of the plantar plate with displacement and joint interposition. Open reduction, most commonly with a dorsal incision and arthrotomy, is required to remove the interposed tissue so the joint will reduce. The joint is then reduced and pinned in place with a k-wire to maintain the reduction. The k-wire pin is kept in place for 4-6 weeks as the joint stabilizes and the surrounding soft tissues heal. An interphalangeal joint fusion may be performed for complex fracture dislocations of the IP joint. 1,2
Answer C.
References
- Brunet JA, Tubin S. Traumatic dislocations of the lesser toes. Foot & ankle international. 1997 Jul;18(7):406-11.
- Córdoba-Fernández A. Management of nonreducible lesser toe interphalangeal dislocation: an unusual injury. Journal of the American Podiatric Medical Association. 2012 May 1;102(3):252-5.
-
Question 34 of 55
34. Question
A 66 year female presents to your office with left hip pain after a fall 10 days ago. She slipped on ice on her driveway and landed on the left hip. She has a history of a left uncemented total hip replacement 10 years ago and before the fall the hip was doing great. She was brought to the hospital by ambulance and xrays showed a periprosthetic fracture of the left hip (Figures 1 and 2). Her pain was well controlled and she was discharged from the ED. Upon follow-up, repeat xrays show no change in alignment of the fracture or the total hip stem (Figures 3 and 4). On physical exam she is sitting comfortably in a wheelchair and has only mild pain with gentle left hip motion. She has no deformity or leg length discrepancy. What is the best treatment option for this patient?
Correct
The patient has a periprosthetic fracture around the intertrochanteric region that doesn’t extend distally past the trochanteric region. The femoral stem appears to be well fixed distally without subsidence. When the stem appears stable with a minimally displaced intertrochanteric fracture, a reasonable approach is to treat nonoperatively with a 8-12 week period of protected weight bearing and avoidance of hip abduction. Fracture extension into the calcar can lead to hip instability so xrays should be followed closely. Periprosthetic osteolysis can form if the hip is unstable which requires operative treatment. If the fracture displaces, or the stem subsides and appears unstable, than cerclage wiring around the lesser trochanter or a revision stem with diaphyseal fixation may be necessary. Trochanteric periprosthetic fractures that are displaced more than 2cm are more likely to progress to a symptomatic nonunion. 1,2
Answer A.
References
- Patsiogiannis N, Kanakaris NK, Giannoudis PV. Periprosthetic hip fractures: an update into their management and clinical outcomes. EFORT Open Reviews. 2021 Jan 4;6(1):75-92.
- Marsland D, Mears SC. A review of periprosthetic femoral fractures associated with total hip arthroplasty. Geriatric orthopaedic surgery & rehabilitation. 2012 Sep;3(3):107-20.
Incorrect
The patient has a periprosthetic fracture around the intertrochanteric region that doesn’t extend distally past the trochanteric region. The femoral stem appears to be well fixed distally without subsidence. When the stem appears stable with a minimally displaced intertrochanteric fracture, a reasonable approach is to treat nonoperatively with a 8-12 week period of protected weight bearing and avoidance of hip abduction. Fracture extension into the calcar can lead to hip instability so xrays should be followed closely. Periprosthetic osteolysis can form if the hip is unstable which requires operative treatment. If the fracture displaces, or the stem subsides and appears unstable, than cerclage wiring around the lesser trochanter or a revision stem with diaphyseal fixation may be necessary. Trochanteric periprosthetic fractures that are displaced more than 2cm are more likely to progress to a symptomatic nonunion. 1,2
Answer A.
References
- Patsiogiannis N, Kanakaris NK, Giannoudis PV. Periprosthetic hip fractures: an update into their management and clinical outcomes. EFORT Open Reviews. 2021 Jan 4;6(1):75-92.
- Marsland D, Mears SC. A review of periprosthetic femoral fractures associated with total hip arthroplasty. Geriatric orthopaedic surgery & rehabilitation. 2012 Sep;3(3):107-20.
-
Question 35 of 55
35. Question
A 15 year old male presents to your office with pain in the small finger after a football injury earlier in the day. He jammed his finger while he was making a tackle and had immediate pain at the base of his small finger. On physical exam he has tenderness to palpation over the proximal phalanx of the small finger without deformity. AP xray of the hand (figure 1) shows a pathological proximal phalanx fracture through what appears to be a bony tumor. What is the best treatment option for this patient?
Correct
The patient has a benign appearing cyst with sclerotic margins which is most consistent with an enchondroma. Enchondromas are the most common benign neoplasm in the hand and occur in the proximal phalanx 50% of the time. Enchondromas are asymptomatic, slow growing tumors and the most common presenting symptom is a pathological fracture. A pathological fracture can result from cortical expansion and thinning from the tumor which inherently weakens the bone. Pathological fractures through enchondromas will generally heal with non-operative treatment including initial treatment in an ulnar gutter cast. An alumafoam splint is used to treat distal and middle phalanx fractures but it doesn’t provide adequate support for proximal phalanx fractures. Pathological fractures through large enchondromas (>50% of the bone width) may need curettage and bone graft to stabilize the bone to prevent re-fracture. This can be done at the initial presentation of the fracture or performed when the fracture heals. 1,2
Answer C
References
1. Lubahn, John D. MD; Bachoura, Abdo MD. Enchondroma of the Hand: Evaluation and Management. Journal of the American Academy of Orthopaedic Surgeons 24(9):p 625-633, September 2016. | DOI: 10.5435/JAAOS-D-15-00452
2. Oflazoglu K, Lans J, Castelein RM, Lozano Calderón SA, Chen NC. Pathologic Fractures in Benign Neoplasms of the Fingers. Hand (N Y). 2021 May;16(3):326-331. doi: 10.1177/1558944719855443. Epub 2019 Jun 17. PMID: 31203664; PMCID: PMC8120596.
Incorrect
The patient has a benign appearing cyst with sclerotic margins which is most consistent with an enchondroma. Enchondromas are the most common benign neoplasm in the hand and occur in the proximal phalanx 50% of the time. Enchondromas are asymptomatic, slow growing tumors and the most common presenting symptom is a pathological fracture. A pathological fracture can result from cortical expansion and thinning from the tumor which inherently weakens the bone. Pathological fractures through enchondromas will generally heal with non-operative treatment including initial treatment in an ulnar gutter cast. An alumafoam splint is used to treat distal and middle phalanx fractures but it doesn’t provide adequate support for proximal phalanx fractures. Pathological fractures through large enchondromas (>50% of the bone width) may need curettage and bone graft to stabilize the bone to prevent re-fracture. This can be done at the initial presentation of the fracture or performed when the fracture heals. 1,2
Answer C
References
1. Lubahn, John D. MD; Bachoura, Abdo MD. Enchondroma of the Hand: Evaluation and Management. Journal of the American Academy of Orthopaedic Surgeons 24(9):p 625-633, September 2016. | DOI: 10.5435/JAAOS-D-15-00452
2. Oflazoglu K, Lans J, Castelein RM, Lozano Calderón SA, Chen NC. Pathologic Fractures in Benign Neoplasms of the Fingers. Hand (N Y). 2021 May;16(3):326-331. doi: 10.1177/1558944719855443. Epub 2019 Jun 17. PMID: 31203664; PMCID: PMC8120596.
-
Question 36 of 55
36. Question
An 18 year old football player presents to your office with right anterior knee pain after taking a helmet to the knee 2 days ago. He was able to continue playing through the injury and finished the game. On examination, he has soft tissue swelling at the anterior knee but no ecchymosis. He has full passive range of motion of the knee and full strength with knee extension. AP, lateral, and merchant view xrays are shown in figures 1, 2, and 3 respectively. What is the most appropriate next step in management?
Correct
A bipartite patella results from the failure of the ossification center to fuse and instead is held together by a fibrocartilaginous interface between the accessory and main patella. Most are asymptomatic and diagnosed incidentally on xray. Rarely a symptomatic bipartite patella may stem from a direct blow to the patella or motion through the fibrocartilaginous interface during exercise. Symptoms are more chronic in nature and associated with knee extension exercises. It is important for providers to recognize a bipartite patella and not jump to an extensive work-up before conservative treatment is trialed. If patients fail conservative treatment an MRI can determine if there is edema around the accessory ossicle indicating a painful bipartite patella. When a symptomatic bipartite patella is diagnosed, conservative treatment is recommended such as knee immobilization, stretching exercises, oral and topical NSAIDs, activity modification. If 6 months of conservative treatment fails surgery may be considered. Surgical options include excision of the accessary fragment, lateral soft tissue release to remove traction forces on the ossicle, or for larger fragments, open reduction and internal fixation. 1,2
Answer A.
References
1. Atesok, Kivanc MD; Doral, Nedim M. MD; Lowe, Joseph MD; Finsterbush, Alex MD. Symptomatic Bipartite Patella: Treatment Alternatives. Journal of the American Academy of Orthopaedic Surgeons 16(8):p 455-461, August 2008.
2. Kallini J, Micheli LJ, Miller PE, Kramer DE, Kocher MS, Heyworth BE. Operative treatment of bipartite patella in pediatric and adolescent athletes: a retrospective comparison with a nonoperatively treated cohort. Orthopaedic Journal of Sports Medicine. 2021 Jan 5;9(1):2325967120967125
Incorrect
A bipartite patella results from the failure of the ossification center to fuse and instead is held together by a fibrocartilaginous interface between the accessory and main patella. Most are asymptomatic and diagnosed incidentally on xray. Rarely a symptomatic bipartite patella may stem from a direct blow to the patella or motion through the fibrocartilaginous interface during exercise. Symptoms are more chronic in nature and associated with knee extension exercises. It is important for providers to recognize a bipartite patella and not jump to an extensive work-up before conservative treatment is trialed. If patients fail conservative treatment an MRI can determine if there is edema around the accessory ossicle indicating a painful bipartite patella. When a symptomatic bipartite patella is diagnosed, conservative treatment is recommended such as knee immobilization, stretching exercises, oral and topical NSAIDs, activity modification. If 6 months of conservative treatment fails surgery may be considered. Surgical options include excision of the accessary fragment, lateral soft tissue release to remove traction forces on the ossicle, or for larger fragments, open reduction and internal fixation. 1,2
Answer A.
References
1. Atesok, Kivanc MD; Doral, Nedim M. MD; Lowe, Joseph MD; Finsterbush, Alex MD. Symptomatic Bipartite Patella: Treatment Alternatives. Journal of the American Academy of Orthopaedic Surgeons 16(8):p 455-461, August 2008.
2. Kallini J, Micheli LJ, Miller PE, Kramer DE, Kocher MS, Heyworth BE. Operative treatment of bipartite patella in pediatric and adolescent athletes: a retrospective comparison with a nonoperatively treated cohort. Orthopaedic Journal of Sports Medicine. 2021 Jan 5;9(1):2325967120967125
-
Question 37 of 55
37. Question
A 59 year old male presents to your office with right shoulder pain for several months. He denies any known injury but the pain is made worse with overhead activities. On physical exam he has mild weakness with rotator cuff testing and positive impingement signs. When looking at his AP and grashey view xrays (figures 1 and 2), what abnormality below is seen?
Correct
The acromion has 4 ossification centers that typically fuse during skeletal maturity or up to 25 years of age. Os acromiale is an uncommon condition that occurs when any of the two ossification centers fail to fuse. The overall incidence of os acromiale is 8% in the general population and most are asymptomatic. Os acromiale can by symptomatic as the lateral edge of the acromion is hypermobile which causes impingement on the rotator cuff. The os acromiale can also develop arthritic changes and become inflamed. MRI is the study of choice to see if there is inflammation in the synchondrosis (unfused ossification center) and to look for rotator cuff pathology. Diagnostic injections can help make the diagnosis including a combination of a failed subacromial injection in conjunction with pain relief from a steroid injection into the synchondrosis. Initial treatment is always observation including rest and NSAIDS unless there is rotator cuff pathology which should be addressed early. If a symptomatic os acromiale is diagnosed, there is no rotator cuff pathology, and conservative treatment for 6 months has failed, surgical options may be necessary. A small os acromiale may be excised or have an arthroscopic decompression where large unstable ones may need open reduction and internal fixation to fuse the joint. 1,2
Answer D.
References
1. Hasan, S. Ashfaq MD; Shiu, Brian MD; Jauregui, Julio J. MD. Symptomatic, Unstable Os Acromiale. Journal of the American Academy of Orthopaedic Surgeons 26(22):p 789-797, November 15, 2018. | DOI: 10.5435/JAAOS-D-17-00011
2. You T, Frostick S, Zhang WT, Yin Q. Os acromiale: reviews and current perspectives. Orthopaedic Surgery. 2019 Oct;11(5):738-44.
Incorrect
The acromion has 4 ossification centers that typically fuse during skeletal maturity or up to 25 years of age. Os acromiale is an uncommon condition that occurs when any of the two ossification centers fail to fuse. The overall incidence of os acromiale is 8% in the general population and most are asymptomatic. Os acromiale can by symptomatic as the lateral edge of the acromion is hypermobile which causes impingement on the rotator cuff. The os acromiale can also develop arthritic changes and become inflamed. MRI is the study of choice to see if there is inflammation in the synchondrosis (unfused ossification center) and to look for rotator cuff pathology. Diagnostic injections can help make the diagnosis including a combination of a failed subacromial injection in conjunction with pain relief from a steroid injection into the synchondrosis. Initial treatment is always observation including rest and NSAIDS unless there is rotator cuff pathology which should be addressed early. If a symptomatic os acromiale is diagnosed, there is no rotator cuff pathology, and conservative treatment for 6 months has failed, surgical options may be necessary. A small os acromiale may be excised or have an arthroscopic decompression where large unstable ones may need open reduction and internal fixation to fuse the joint. 1,2
Answer D.
References
1. Hasan, S. Ashfaq MD; Shiu, Brian MD; Jauregui, Julio J. MD. Symptomatic, Unstable Os Acromiale. Journal of the American Academy of Orthopaedic Surgeons 26(22):p 789-797, November 15, 2018. | DOI: 10.5435/JAAOS-D-17-00011
2. You T, Frostick S, Zhang WT, Yin Q. Os acromiale: reviews and current perspectives. Orthopaedic Surgery. 2019 Oct;11(5):738-44.
-
Question 38 of 55
38. Question
A 16 year old male presents to your office with left ankle pain after a football injury. He was tackled while running and his ankle rolled underneath him. He had immediate pain and was unable to bear weight. On physical exam he has diffuse swelling throughout the ankle without deformity. He has tenderness to palpation over the anterior syndesmosis, the medial deltoid ligament, and the distal lateral malleolus. AP and lateral xrays of the left ankle show lateral and posterior malleoli fractures with subtle medial clear space widening (Figures 1 and 2). Which treatment option offers the patient the earliest return to sports with the best long term outcome?
Correct
The ankle syndesmosis is made of strong ligaments that stabilize the ankle including the anterior, posterior, and interosseous ligaments between the tibia and the fibular. The syndesmosis maintains stability of the ankle joint against axial and rotational forces. Injury to the syndesmosis and medial deltoid ligament causes tibial-fibular diastasis which increases the contact pressures on the talus. Lateral displacement of the talus also increases contact pressure in the ankle. Over time the increased contact pressures and instability of the ankle can result in early degenerative changes. Evidence of an unstable syndesmotic injury includes pain to palpation over the anterior syndesmosis, pain to palpation over the deltoid ligament (indicating a deltoid tear), and increase medial clear space widening (with or without stress radiographs). A posterior malleolus fracture further destabilizes the syndesmosis as the deep posterior inferior tibiofibular ligament provides 32.7% of the ligamentous resistance of the syndesmosis. This patient has medial clear space widening and tenderness to palpation over the deltoid ligament indicating an unstable ankle injury. Surgical treatment is recommended for unstable syndesmotic injuries to restore the syndesmosis to an anatomical position. Open reduction and internal fixation of the fibular and syndesmotic repair with a screw or suture button is performed. Suture button repair offers an earlier return to activities and improved ankle function over screw fixation. 1,2
Answer B.
References
1. Ryan, Paul M. MD, FAAOS; Eakin, John L. MD; Goodrum, Jason T. MD. Subtle Syndesmotic Instability. Journal of the American Academy of Orthopaedic Surgeons 32(16):p 719-727, August 15, 2024. | DOI: 10.5435/JAAOS-D-23-00707
2. Jones CB, Glide A, Sietsema DL. Treatment of Syndesmotic Injuries of the Ankle. JBJS Rev, 3(10):e1 | Review Article with Critical Analysis Component | October 06, 2015
Incorrect
The ankle syndesmosis is made of strong ligaments that stabilize the ankle including the anterior, posterior, and interosseous ligaments between the tibia and the fibular. The syndesmosis maintains stability of the ankle joint against axial and rotational forces. Injury to the syndesmosis and medial deltoid ligament causes tibial-fibular diastasis which increases the contact pressures on the talus. Lateral displacement of the talus also increases contact pressure in the ankle. Over time the increased contact pressures and instability of the ankle can result in early degenerative changes. Evidence of an unstable syndesmotic injury includes pain to palpation over the anterior syndesmosis, pain to palpation over the deltoid ligament (indicating a deltoid tear), and increase medial clear space widening (with or without stress radiographs). A posterior malleolus fracture further destabilizes the syndesmosis as the deep posterior inferior tibiofibular ligament provides 32.7% of the ligamentous resistance of the syndesmosis. This patient has medial clear space widening and tenderness to palpation over the deltoid ligament indicating an unstable ankle injury. Surgical treatment is recommended for unstable syndesmotic injuries to restore the syndesmosis to an anatomical position. Open reduction and internal fixation of the fibular and syndesmotic repair with a screw or suture button is performed. Suture button repair offers an earlier return to activities and improved ankle function over screw fixation. 1,2
Answer B.
References
1. Ryan, Paul M. MD, FAAOS; Eakin, John L. MD; Goodrum, Jason T. MD. Subtle Syndesmotic Instability. Journal of the American Academy of Orthopaedic Surgeons 32(16):p 719-727, August 15, 2024. | DOI: 10.5435/JAAOS-D-23-00707
2. Jones CB, Glide A, Sietsema DL. Treatment of Syndesmotic Injuries of the Ankle. JBJS Rev, 3(10):e1 | Review Article with Critical Analysis Component | October 06, 2015
-
Question 39 of 55
39. Question
A 25 year old female presents with left foot pain in her first ray for the last 5 months. She does not recall any specific trauma to the foot but enjoys running and yoga. Over the last few months rest and the use of a metatarsal bar orthotic have not helped. Physical exam of the left foot reveals no obvious deformity and she can toe and heel walk without difficulty. She is tender along the lateral sesamoid plantarly. She has full range of motion of the ankle and the first metatarsophalangeal joint. AP and oblique xrays (figures 1 and 2) of the great toe show irregularity to the lateral sesamoid. MRI (figure 3) shows a fracture of the fibular sesamoid with edema. What is the best treatment option for this patient?
Correct
The great toe has two sesamoid bones (tibial sesamoid and fibular sesamoid) on the plantar side of the foot which help provide a mechanical advantage to flexion of the great toe. The tibial sesamoid on the medial part of the foot is larger and more prone to injury. The differential diagnosis for great toe pain is broad so sesamoiditis is often a delayed diagnosis. Activities that cause sesamoid pain may include running, ballet dancing, and jumping from a height. Xrays, including a sesamoid view, should be obtained first if patients have pain over the sesamoids. An acute fracture or bipartite sesamoid may be visible on xray. MRI is the imaging modality of choice for sesamoid pathology and can distinguish between a bipartite sesamoid and a fracture non-union. Conservative treatment is initiated first to unload the sesamoids including a metatarsal bar, a weight bearing walking boot, and activity modification. Acute fractures should be immobilized in a walking boot for 6-8 weeks. Steroid injections should be used with caution with sesamoiditis and avoided with a fracture. When conservative treatments fail sesamoid excision offers a reliable treatment option for pain relief. 1,2
Answer A.
References
1. Dedmond, Barnaby T. MD; Cory, John W. MD; McBryde, Angus Jr MD, FACS. The Hallucal Sesamoid Complex. Journal of the American Academy of Orthopaedic Surgeons 14(13):p 745-753, December 2006.
2. Sims AL, Kurup HV. Painful sesamoid of the great toe. World J Orthop. 2014 Apr 18;5(2):146-50. doi: 10.5312/wjo.v5.i2.146. PMID: 24829877; PMCID: PMC4017307.
Incorrect
The great toe has two sesamoid bones (tibial sesamoid and fibular sesamoid) on the plantar side of the foot which help provide a mechanical advantage to flexion of the great toe. The tibial sesamoid on the medial part of the foot is larger and more prone to injury. The differential diagnosis for great toe pain is broad so sesamoiditis is often a delayed diagnosis. Activities that cause sesamoid pain may include running, ballet dancing, and jumping from a height. Xrays, including a sesamoid view, should be obtained first if patients have pain over the sesamoids. An acute fracture or bipartite sesamoid may be visible on xray. MRI is the imaging modality of choice for sesamoid pathology and can distinguish between a bipartite sesamoid and a fracture non-union. Conservative treatment is initiated first to unload the sesamoids including a metatarsal bar, a weight bearing walking boot, and activity modification. Acute fractures should be immobilized in a walking boot for 6-8 weeks. Steroid injections should be used with caution with sesamoiditis and avoided with a fracture. When conservative treatments fail sesamoid excision offers a reliable treatment option for pain relief. 1,2
Answer A.
References
1. Dedmond, Barnaby T. MD; Cory, John W. MD; McBryde, Angus Jr MD, FACS. The Hallucal Sesamoid Complex. Journal of the American Academy of Orthopaedic Surgeons 14(13):p 745-753, December 2006.
2. Sims AL, Kurup HV. Painful sesamoid of the great toe. World J Orthop. 2014 Apr 18;5(2):146-50. doi: 10.5312/wjo.v5.i2.146. PMID: 24829877; PMCID: PMC4017307.
-
Question 40 of 55
40. Question
A 38 year old male presents to your office with left knee pain and swelling for 6 months. He believes the knee pain started after a fall but the history is unclear. Over the last few months he feels like the knee “catches” and gives out randomly when he is walking. On physical exam he has a large effusion but only mild pain with full passive range of motion. He has no medial or lateral joint line pain. He has no instability with varus and valgus stress to the knee and a negative Lachman’s. On AP and lateral view xrays you see a bone fragment in the posterior aspect of the knee (see white arrows). What is the best treatment for this bone fragment?
Correct
The patient has a fabella which is highlighted by the white arrows. The fabella is an incidental finding and asymptomatic. No treatment is necessary for an asymptomatic fabella. On closer inspection of the lateral xray there is also a loose body at the medial femoral condyle extending into the intercondylar notch which most likely represents a loose osteochondral fragment that is causing his symptoms.
The fabella is a sesamoid bone that is embedded in the lateral gastrocnemius muscle and is rarely associated with pain or pathology. The fabella can be best seen on lateral radiographs and occurs in 10-20% of the population. Fabella syndrome can occur with physical activities where the sesamoid and surrounding soft tissues become inflamed with compression against the lateral femoral condyle. This most commonly occurs in teenagers and is characterized by posterolateral knee pain with knee extension and tenderness to palpation over the posterolateral gastrocnemius. MRI may show thickening of the lateral gastrocnemius tendon, inflammation of the fabella, and grooving of the lateral femoral condyle articular cartilage due to the articulating fabella. Fabella syndrome is often a diagnosis of exclusion. Local steroid injections can help confirm the diagnosis if relief is achieved post-injection. Fabella excision is a last resort surgical option if patients fail several months of conservative treatment. 1,2
Answer A.
References
1. Lustig M, Hazzard S. Fabella Syndrome of the Knee. An Overview on Diagnosis and Management of an Uncommon Condition. JBJS JOPA, 11(2):e23.00004 | Review Articles | June 06, 2023
2. Dekker TJ, Crawford MD, DePhillipo NN, Kennedy MI, Grantham WJ, Schairer WW, LaPrade RF. Clinical presentation and outcomes associated with fabellectomy in the setting of fabella syndrome. Orthopaedic Journal of Sports Medicine. 2020 Feb 25;8(2):2325967120903722.
Incorrect
The patient has a fabella which is highlighted by the white arrows. The fabella is an incidental finding and asymptomatic. No treatment is necessary for an asymptomatic fabella. On closer inspection of the lateral xray there is also a loose body at the medial femoral condyle extending into the intercondylar notch which most likely represents a loose osteochondral fragment that is causing his symptoms.
The fabella is a sesamoid bone that is embedded in the lateral gastrocnemius muscle and is rarely associated with pain or pathology. The fabella can be best seen on lateral radiographs and occurs in 10-20% of the population. Fabella syndrome can occur with physical activities where the sesamoid and surrounding soft tissues become inflamed with compression against the lateral femoral condyle. This most commonly occurs in teenagers and is characterized by posterolateral knee pain with knee extension and tenderness to palpation over the posterolateral gastrocnemius. MRI may show thickening of the lateral gastrocnemius tendon, inflammation of the fabella, and grooving of the lateral femoral condyle articular cartilage due to the articulating fabella. Fabella syndrome is often a diagnosis of exclusion. Local steroid injections can help confirm the diagnosis if relief is achieved post-injection. Fabella excision is a last resort surgical option if patients fail several months of conservative treatment. 1,2
Answer A.
References
1. Lustig M, Hazzard S. Fabella Syndrome of the Knee. An Overview on Diagnosis and Management of an Uncommon Condition. JBJS JOPA, 11(2):e23.00004 | Review Articles | June 06, 2023
2. Dekker TJ, Crawford MD, DePhillipo NN, Kennedy MI, Grantham WJ, Schairer WW, LaPrade RF. Clinical presentation and outcomes associated with fabellectomy in the setting of fabella syndrome. Orthopaedic Journal of Sports Medicine. 2020 Feb 25;8(2):2325967120903722.
-
Question 41 of 55
41. Question
A 60 year old male presents to your office with lesser toe pain for several months. He is getting pain and deformity of the second toe which is rubbing against his great toe. He has tried silicone sleeves to the second toe and shoes with a tall toe box with mild relief for the last 4 months. On physical exam he has rigid second and third hammertoes and a flexible fourth hammertoe. Figure 1 shows a picture of the lesser toes. Xrays of the foot reveal hammering of the second and third toes and degenerative changes in the first metatarsophalangeal joint. What is the best treatment option for this patient?
Correct
Lesser toe deformities (mallet toes, hammer toes, and claw toes) are common conditions seen in orthopedic practice. Hammer toe is characterized by a flexion deformity at the PIP joint, extension at the DIP joint, and an extended or neutral MTP joint. There is a higher incidence in women, likely due to shoe ware and the longer lesser toe is the most affected. Hammertoes can become symptomatic when the dorsal PIP joint rubs against shoe ware, often causing a painful hypertrophic callus. The second toe can also cross over or crowd the big toe, particularly with a hallux valgus deformity, causing painful rubbing between the toes. On physical exam, flexion of the PIP joint can be either fixed, semiflexible, or flexible, which helps determine treatment. Conservative treatment involves taking pressure off the dorsal PIP joint with high toe boxes and foam or gel sleeves. For surgical options, a flexible deformity can be managed with a flexor digitorum longus tendon transfer, but fixed deformity requires PIP resection arthroplasty or fusion.1,2
Answer B.
References
1. Shirzad, Khalid MD; Kiesau, Carter D. MD; DeOrio, James K. MD; Parekh, Selene G. MD, MBA. Lesser Toe Deformities. American Academy of Orthopaedic Surgeon 19(8):p 505-514, August 2011.
2. Image Quiz: Hammer Toe. JBJS JOPA, 5(4):e25 | Image Quiz | October 04, 2017
Incorrect
Lesser toe deformities (mallet toes, hammer toes, and claw toes) are common conditions seen in orthopedic practice. Hammer toe is characterized by a flexion deformity at the PIP joint, extension at the DIP joint, and an extended or neutral MTP joint. There is a higher incidence in women, likely due to shoe ware and the longer lesser toe is the most affected. Hammertoes can become symptomatic when the dorsal PIP joint rubs against shoe ware, often causing a painful hypertrophic callus. The second toe can also cross over or crowd the big toe, particularly with a hallux valgus deformity, causing painful rubbing between the toes. On physical exam, flexion of the PIP joint can be either fixed, semiflexible, or flexible, which helps determine treatment. Conservative treatment involves taking pressure off the dorsal PIP joint with high toe boxes and foam or gel sleeves. For surgical options, a flexible deformity can be managed with a flexor digitorum longus tendon transfer, but fixed deformity requires PIP resection arthroplasty or fusion.1,2
Answer B.
References
1. Shirzad, Khalid MD; Kiesau, Carter D. MD; DeOrio, James K. MD; Parekh, Selene G. MD, MBA. Lesser Toe Deformities. American Academy of Orthopaedic Surgeon 19(8):p 505-514, August 2011.
2. Image Quiz: Hammer Toe. JBJS JOPA, 5(4):e25 | Image Quiz | October 04, 2017
-
Question 42 of 55
42. Question
A 22 year old female presents to your office with left foot pain after twisting the foot during a tackle in a rugby match 2 days ago. She had immediate pain and was unable to continue playing. On physical exam she has mild swelling to the midfoot and pain to palpation over the medial navicular. AP and lateral xrays of the foot show a nondisplaced navicular tuberosity fracture (figures 1 and 2.) What is the best treatment option?
Correct
The tarsal navicular is a saucer shaped bone that articulates proximally with the talus, laterally with the cuboid, and distally with the 3 cuneiforms. Much of the bone is covered with cartilage and the blood supply can be tenuous leading to an increased risk of avascular necrosis and fracture nonunion. The talonavicular joint is responsible for most hindfoot motion and the talus, navicular, and medial cuneiform form the medial column of the foot. The medial column and the posterior tibial tendon (which attaches to the navicular) form the medial arch which stabilizes the foot during ambulation. The tarsal navicular is the most common tarsal bone fractured. Severity of fractures can vary widely from the common dorsal avulsion fracture (50% of all navicular fractures), stress fractures, to higher energy navicular body fractures. Dorsal navicular avulsion fractures can be treated in a walking boot weightbearing to tolerance for 4-6 weeks with a gradual return to sports thereafter. Navicular stress fractures and non-displaced navicular body fractures should be treated with a non-weight bearing boot or cast for 6 weeks. 1,2
Answer C.
References
1. Ramadorai, MAJ Uma E. DO; Beuchel, Matthew W. MD; Sangeorzan, Bruce J. MD. Fractures and Dislocations of the Tarsal Navicular. Journal of the American Academy of Orthopaedic Surgeons 24(6):p 379-389, June 2016. | DOI: 10.5435/JAAOS-D-14-00442
2. Rosenbaum AJ, Uhl RL, DiPreta JA. Acute fractures of the tarsal navicular. Orthopedics. 2014 Aug 1;37(8):541-6.
Incorrect
The tarsal navicular is a saucer shaped bone that articulates proximally with the talus, laterally with the cuboid, and distally with the 3 cuneiforms. Much of the bone is covered with cartilage and the blood supply can be tenuous leading to an increased risk of avascular necrosis and fracture nonunion. The talonavicular joint is responsible for most hindfoot motion and the talus, navicular, and medial cuneiform form the medial column of the foot. The medial column and the posterior tibial tendon (which attaches to the navicular) form the medial arch which stabilizes the foot during ambulation. The tarsal navicular is the most common tarsal bone fractured. Severity of fractures can vary widely from the common dorsal avulsion fracture (50% of all navicular fractures), stress fractures, to higher energy navicular body fractures. Dorsal navicular avulsion fractures can be treated in a walking boot weightbearing to tolerance for 4-6 weeks with a gradual return to sports thereafter. Navicular stress fractures and non-displaced navicular body fractures should be treated with a non-weight bearing boot or cast for 6 weeks. 1,2
Answer C.
References
1. Ramadorai, MAJ Uma E. DO; Beuchel, Matthew W. MD; Sangeorzan, Bruce J. MD. Fractures and Dislocations of the Tarsal Navicular. Journal of the American Academy of Orthopaedic Surgeons 24(6):p 379-389, June 2016. | DOI: 10.5435/JAAOS-D-14-00442
2. Rosenbaum AJ, Uhl RL, DiPreta JA. Acute fractures of the tarsal navicular. Orthopedics. 2014 Aug 1;37(8):541-6.
-
Question 43 of 55
43. Question
A 14 year old male presents the ED with elbow pain and deformity after a fall from a trampoline. Initial xrays of the left elbow show a posterior elbow dislocation (Figures 1 and 2). The patient was closed reduced in the ED under conscious sedation. When putting on a posterior long arm splint you notice a lack of full extension to the left elbow. Post reduction xrays are shown in (Figures 3 and 4). What is the next best step in treatment?
Correct
The patient has an incarcerated medial epicondyle apophyseal avulsion fracture after closed reduction of the elbow. The medial epicondyle apophysis is the last ossification center to fuse in the distal humerus, generally around 15 to 20 years of age. The medial epicondyle is the attachment site for the flexor pronator mass. The most common fracture of the elbow that results from a pediatric elbow dislocation is a medial epicondyle avulsion fracture. The avulsed medial epicondyle can become incarcerated in the elbow joint which can cause devastating consequences if not recognized. Up to 25% of medial epicondyle fractures will become incarcerated after closed reduction of an elbow dislocation. Signs of an incarcerated medial condyle fracture include a block to motion (most commonly a lack of full elbow extension) and xray findings of the apophysis at the joint level. The fracture can spontaneously reduce itself so a careful exam and review of post reduction xrays are critical. An incarcerated medial epicondyle fracture after closed reduction is an indication for an urgent open reduction and internal fixation. It is also critical to recognize ulna nerve dysfunction after an elbow dislocation which would require an ulnar nerve exploration during operative fixation. 1,2
Answer D.
References
1. Dodds SD, Flanagin BA, Bohl DD, DeLuca PA, Smith BG. Incarcerated medial epicondyle fracture following pediatric elbow dislocation: 11 cases. The Journal of Hand Surgery. 2014 Sep 1;39(9):1739-45.
2. Gottschalk, Hilton P. MD; Eisner, Eric MD; Hosalkar, Harish S. MD. Medial Epicondyle Fractures in the Pediatric Population. Journal of the American Academy of Orthopaedic Surgeons 20(4):p 223-232, April 2012. | DOI: 10.5435/JAAOS-20-04-223
Incorrect
The patient has an incarcerated medial epicondyle apophyseal avulsion fracture after closed reduction of the elbow. The medial epicondyle apophysis is the last ossification center to fuse in the distal humerus, generally around 15 to 20 years of age. The medial epicondyle is the attachment site for the flexor pronator mass. The most common fracture of the elbow that results from a pediatric elbow dislocation is a medial epicondyle avulsion fracture. The avulsed medial epicondyle can become incarcerated in the elbow joint which can cause devastating consequences if not recognized. Up to 25% of medial epicondyle fractures will become incarcerated after closed reduction of an elbow dislocation. Signs of an incarcerated medial condyle fracture include a block to motion (most commonly a lack of full elbow extension) and xray findings of the apophysis at the joint level. The fracture can spontaneously reduce itself so a careful exam and review of post reduction xrays are critical. An incarcerated medial epicondyle fracture after closed reduction is an indication for an urgent open reduction and internal fixation. It is also critical to recognize ulna nerve dysfunction after an elbow dislocation which would require an ulnar nerve exploration during operative fixation. 1,2
Answer D.
References
1. Dodds SD, Flanagin BA, Bohl DD, DeLuca PA, Smith BG. Incarcerated medial epicondyle fracture following pediatric elbow dislocation: 11 cases. The Journal of Hand Surgery. 2014 Sep 1;39(9):1739-45.
2. Gottschalk, Hilton P. MD; Eisner, Eric MD; Hosalkar, Harish S. MD. Medial Epicondyle Fractures in the Pediatric Population. Journal of the American Academy of Orthopaedic Surgeons 20(4):p 223-232, April 2012. | DOI: 10.5435/JAAOS-20-04-223
-
Question 44 of 55
44. Question
A 12 year old male presents the office with right wrist pain after a fall 2 days ago. He was playing soccer and fell on the wrist with a direct fall on an outstretched hand. He had immediate pain but no deformity. The pain and swelling increased and now he can’t use the wrist without pain. AP, oblique, and lateral xrays show a minimally displaced ephiphyseal fracture (figures 1, 2, and 3). What is the best treatment option to help prevent growth arrest?
Correct
Distal radius fractures are the most common pediatric injury accounting for up to 30% of all fractures in the pediatric population. Growth plate injury and growth arrest are rare after distal radius fractures with a 1% to 7% incidence after type 1 and II Salter Harris fractures. Growth arrest has a higher incidence with repeated forceful reductions and closed reduction performed 7-10 days after injury. For this reason, patients with greater than 2 years of growth remaining, no reduction should be performed initially, or if the fracture displaces after 7-10 days from injury. Fractures around the physis have excellent remodeling potential so epiphyseal fractures with less than 50% displacement and acceptable angulation can be treated with immobilization alone. Epiphyseal fractures should be immobilized in a cast for 6 weeks with serial radiographic follow-up. A follow-up xray can be performed at 6 months after the fracture is healed to assess the physeal plate. Physeal injury can be seen as a physeal bar or a prominent ulnar variance. Growth plate abnormalities are more common in Salter Harris III and IV injuries. 1,2
Answer C.
References
1. Abzug, Joshua M. MD; Little, Kevin MD; Kozin, Scott H. MD. Physeal Arrest of the Distal Radius. Journal of the American Academy of Orthopaedic Surgeons 22(6):p 381-389, June 2014. | DOI: 10.5435/JAAOS-22-06-381
2. Wilkins, Kaye E. Epiphyseal Growth Plate Fractures. J Bone Joint Surg Am, 90(1):223-224 | Book Reviews | January 01, 2008
Incorrect
Distal radius fractures are the most common pediatric injury accounting for up to 30% of all fractures in the pediatric population. Growth plate injury and growth arrest are rare after distal radius fractures with a 1% to 7% incidence after type 1 and II Salter Harris fractures. Growth arrest has a higher incidence with repeated forceful reductions and closed reduction performed 7-10 days after injury. For this reason, patients with greater than 2 years of growth remaining, no reduction should be performed initially, or if the fracture displaces after 7-10 days from injury. Fractures around the physis have excellent remodeling potential so epiphyseal fractures with less than 50% displacement and acceptable angulation can be treated with immobilization alone. Epiphyseal fractures should be immobilized in a cast for 6 weeks with serial radiographic follow-up. A follow-up xray can be performed at 6 months after the fracture is healed to assess the physeal plate. Physeal injury can be seen as a physeal bar or a prominent ulnar variance. Growth plate abnormalities are more common in Salter Harris III and IV injuries. 1,2
Answer C.
References
1. Abzug, Joshua M. MD; Little, Kevin MD; Kozin, Scott H. MD. Physeal Arrest of the Distal Radius. Journal of the American Academy of Orthopaedic Surgeons 22(6):p 381-389, June 2014. | DOI: 10.5435/JAAOS-22-06-381
2. Wilkins, Kaye E. Epiphyseal Growth Plate Fractures. J Bone Joint Surg Am, 90(1):223-224 | Book Reviews | January 01, 2008
-
Question 45 of 55
45. Question
A 45 year old male presents to your clinic with ongoing deep-seated right anterior hip discomfort for the past 2 years. He denies a known injury or precipitating event. He notices the pain when he is running and with hip rotation. He has tried physical therapy and a 3-month period of rest without significant relief. He denies any family history or childhood history of hip problem. He feels like his hip “pops” on occasion when he plays tennis. Axial MRI image of the hip shows a small anterior superior labral tear with paralabral cyst (figure 1). There is also evidence of Cam and Pincer lesions with moderate osteoarthritis. The patient is considering hip arthroscopy. Which concomitant condition is the most important predictor of long-term success after arthroscopic labral repair?
Correct
Hip arthroscopy with labral repair is an increasingly used surgical procedure that offers excellent results in the right patients. The indications for a labral repair include persistent hip pain despite a 3-6 month trial of conservative treatment and MRI confirmation of a labral tear. Labral tears are very common (found in 69% of the asymptomatic population of patients aged 15 to 66 years) so confirming the labrum as the source of pain is critical. Cam and pincer lesion are bony growths on the femoral neck and acetabulum, respectively, which cause femoroacetabular impingement and associated labral tears. Cam and pincer lesions are addressed at the time of labral repair with a femoroplasty and acetabuloplasty (arthroscopic removal of the bony prominences). Advanced osteoarthritis of the hip is a contra-indication for labral repair. Patients with advanced arthritis are much more likely to fail labral repair and labral debridement and require a total hip arthroplasty within 5-6 years of hip arthroscopy. Residual femoroacetabular impingement and acetabular dysplasia are also significant predictors of failure of labral repair. 1,2
Answer B.
References
1. Domb, Benjamin G. MD; Hartigan, David E. MD; Perets, Itay MD. Decision Making for Labral Treatment in the Hip: Repair Versus Débridement Versus Reconstruction. Journal of the American Academy of Orthopaedic Surgeons 25(3):p e53-e62, March 2017. | DOI: 10.5435/JAAOS-D-16-00144
2. Harris JD. Hip labral repair: options and outcomes. Current reviews in musculoskeletal medicine. 2016 Dec;9(4):361-7.
Incorrect
Hip arthroscopy with labral repair is an increasingly used surgical procedure that offers excellent results in the right patients. The indications for a labral repair include persistent hip pain despite a 3-6 month trial of conservative treatment and MRI confirmation of a labral tear. Labral tears are very common (found in 69% of the asymptomatic population of patients aged 15 to 66 years) so confirming the labrum as the source of pain is critical. Cam and pincer lesion are bony growths on the femoral neck and acetabulum, respectively, which cause femoroacetabular impingement and associated labral tears. Cam and pincer lesions are addressed at the time of labral repair with a femoroplasty and acetabuloplasty (arthroscopic removal of the bony prominences). Advanced osteoarthritis of the hip is a contra-indication for labral repair. Patients with advanced arthritis are much more likely to fail labral repair and labral debridement and require a total hip arthroplasty within 5-6 years of hip arthroscopy. Residual femoroacetabular impingement and acetabular dysplasia are also significant predictors of failure of labral repair. 1,2
Answer B.
References
1. Domb, Benjamin G. MD; Hartigan, David E. MD; Perets, Itay MD. Decision Making for Labral Treatment in the Hip: Repair Versus Débridement Versus Reconstruction. Journal of the American Academy of Orthopaedic Surgeons 25(3):p e53-e62, March 2017. | DOI: 10.5435/JAAOS-D-16-00144
2. Harris JD. Hip labral repair: options and outcomes. Current reviews in musculoskeletal medicine. 2016 Dec;9(4):361-7.
-
Question 46 of 55
46. Question
A 32 year old female presents to your office with right hip pain for the last 6 months. She describes the pain as deep-seated lateral hip, medial thigh, and groin pain. She tried a few months of physical therapy which offered some relief but her symptoms remain. AP xray of the pelvis (figure 1) shows no evidence of arthritis or acute abnormalities. Three months ago she underwent an MR arthrogram (figure 2) showing an anterior superior labral tear. At the time of arthrogram she had an intra-articular steroid injection which relieved all of her pain for a few days. On exam, she has worsening pain when her hip is brought into flexion, adduction and internal rotation. Which part of the patient’s history offers the best prognostic indicator that a hip arthroscopy with labral repair would offer the patient long term relief?
Correct
The high prevalence of asymptomatic labral tears in the young adult population can be a challenging dilemma when identifying the source of hip pain. One study found labral tears in up to 69% of asymptomatic population between 15 and 66 years of age, where another study found the prevalence as high as 89% in the general population. Making an accurate diagnosis relies heavily on clinical exam findings (such as a positive flexion-adduction-internal rotation test) and a history of painful locking or clicking of the hip with athletic activities. MRI arthrogram is the diagnostic study of choice to identify a labral tear. The arthrogram involves injection of a contrast material under fluoroscopic guidance. At the time of arthrogram, an injection of steroid into the hip can offer a valuable diagnostic and therapeutic option. The steroid can be injected through the same needle as the arthrogram so both can be done at the same time without the additional needle stick. An intra-articular steroid injection that provides relief of hip pain also offers a prognostic indicator that the pain is indeed intra-articular and a hip arthroscopy is more likely to be successful. The only draw back of an intra-articular injection is an increased risk of infection after hip arthroscopy if the injection is done within a month of surgery. For this reason, surgery is generally delayed 1-3 months after the steroid injection if labral repair is deemed necessary. 1,2
Answer B.
References
1. Domb, Benjamin G. MD; Hartigan, David E. MD; Perets, Itay MD. Decision Making for Labral Treatment in the Hip: Repair Versus Débridement Versus Reconstruction. Journal of the American Academy of Orthopaedic Surgeons 25(3):p e53-e62, March 2017. | DOI: 10.5435/JAAOS-D-16-00144
2. Sarna, Neil MD; Onor, Gabriel I. Jr MD; Schenker, Kathleen E. MD; Su, Alvin W. MD, PhD. Combined Single-Shot Intra-articular Injection of Ropivacaine and Contrast Media for Magnetic Resonance Imaging: Surgical Decision Making for Hip Arthroscopy in Young Patients. JAAOS: Global Research and Reviews 8(11):e24.00305, November 2024. | DOI: 10.5435/JAAOSGlobal-D-24-00305
Incorrect
The high prevalence of asymptomatic labral tears in the young adult population can be a challenging dilemma when identifying the source of hip pain. One study found labral tears in up to 69% of asymptomatic population between 15 and 66 years of age, where another study found the prevalence as high as 89% in the general population. Making an accurate diagnosis relies heavily on clinical exam findings (such as a positive flexion-adduction-internal rotation test) and a history of painful locking or clicking of the hip with athletic activities. MRI arthrogram is the diagnostic study of choice to identify a labral tear. The arthrogram involves injection of a contrast material under fluoroscopic guidance. At the time of arthrogram, an injection of steroid into the hip can offer a valuable diagnostic and therapeutic option. The steroid can be injected through the same needle as the arthrogram so both can be done at the same time without the additional needle stick. An intra-articular steroid injection that provides relief of hip pain also offers a prognostic indicator that the pain is indeed intra-articular and a hip arthroscopy is more likely to be successful. The only draw back of an intra-articular injection is an increased risk of infection after hip arthroscopy if the injection is done within a month of surgery. For this reason, surgery is generally delayed 1-3 months after the steroid injection if labral repair is deemed necessary. 1,2
Answer B.
References
1. Domb, Benjamin G. MD; Hartigan, David E. MD; Perets, Itay MD. Decision Making for Labral Treatment in the Hip: Repair Versus Débridement Versus Reconstruction. Journal of the American Academy of Orthopaedic Surgeons 25(3):p e53-e62, March 2017. | DOI: 10.5435/JAAOS-D-16-00144
2. Sarna, Neil MD; Onor, Gabriel I. Jr MD; Schenker, Kathleen E. MD; Su, Alvin W. MD, PhD. Combined Single-Shot Intra-articular Injection of Ropivacaine and Contrast Media for Magnetic Resonance Imaging: Surgical Decision Making for Hip Arthroscopy in Young Patients. JAAOS: Global Research and Reviews 8(11):e24.00305, November 2024. | DOI: 10.5435/JAAOSGlobal-D-24-00305
-
Question 47 of 55
47. Question
A 53 year old male presents to your office with left small finger pain and interphalangeal joint contracture for 4 months. He said he was originally seen for a middle phalanx fracture 4 months ago and was placed in a finger splint. He decided not to follow-up with orthopedics and self-treated himself by keeping the splint on the finger for 8 weeks. When he took the splint off the finger felt like it was stuck in flexion which seems to have become more painful and stiff since. On physical exam the small finger interphalangeal joint is stuck in 45 degrees of flexion and cannot be flexed or extended beyond this position. Any attempts at passive flexion or extension is painful. Figure 1 and 2 include AP and lateral x rays of the hand. Figures 3 and 4 are pictures of the finger showing the small finger flexion contracture. Which option would offer this patient the best improvement in joint motion?
Correct
The patient is presenting with a flexion contracture of the proximal interphalangeal (PIP) joint. PIP joint contractures can be disabling by reducing grip strength, making it difficult to shake hands or put on gloves, and painful to use in general. This patients fixed flexion deformity was due to prolonged immobilization of the PIP joint in flexion which caused capsular and ligamentous contractures. Adhesions of the flexor tendons at the site of fracture can also contribute to the flexion deformity. For this reason early joint motion is recommended with finger fractures and the interphalangeal joint should not be immobilized beyond 3 weeks. A trial of serial casting and dynamic splinting can be used but improvements in motion are often minimal and recurrence of the flexion contracture is common. Surgery with an open release offers the most benefit with an average increase in 30 degrees of extension. Succes of an open release is often determined by patient compliance with postoperative therapy. The last option includes an IP joint fusion or amputation when range of motion can’t be restored. 1,2
Answer C.
1. Catalano, Louis W. III MD; Barron, O. Alton MD; Glickel, Steven Z. MD; Minhas, Shobhit V. MD. Etiology, Evaluation, and Management Options for the Stiff Digit. Journal of the American Academy of Orthopaedic Surgeons 27(15):p e676-e684, August 1, 2019. | DOI: 10.5435/JAAOS-D-18-00310
2. Hogan, Christopher J. MD; Nunley, James A. MD. Posttraumatic Proximal Interphalangeal Joint Flexion Contractures. Journal of the American Academy of Orthopaedic Surgeons 14(9):p 524-533, September 2006.
Incorrect
The patient is presenting with a flexion contracture of the proximal interphalangeal (PIP) joint. PIP joint contractures can be disabling by reducing grip strength, making it difficult to shake hands or put on gloves, and painful to use in general. This patients fixed flexion deformity was due to prolonged immobilization of the PIP joint in flexion which caused capsular and ligamentous contractures. Adhesions of the flexor tendons at the site of fracture can also contribute to the flexion deformity. For this reason early joint motion is recommended with finger fractures and the interphalangeal joint should not be immobilized beyond 3 weeks. A trial of serial casting and dynamic splinting can be used but improvements in motion are often minimal and recurrence of the flexion contracture is common. Surgery with an open release offers the most benefit with an average increase in 30 degrees of extension. Succes of an open release is often determined by patient compliance with postoperative therapy. The last option includes an IP joint fusion or amputation when range of motion can’t be restored. 1,2
Answer C.
1. Catalano, Louis W. III MD; Barron, O. Alton MD; Glickel, Steven Z. MD; Minhas, Shobhit V. MD. Etiology, Evaluation, and Management Options for the Stiff Digit. Journal of the American Academy of Orthopaedic Surgeons 27(15):p e676-e684, August 1, 2019. | DOI: 10.5435/JAAOS-D-18-00310
2. Hogan, Christopher J. MD; Nunley, James A. MD. Posttraumatic Proximal Interphalangeal Joint Flexion Contractures. Journal of the American Academy of Orthopaedic Surgeons 14(9):p 524-533, September 2006.
-
Question 48 of 55
48. Question
A 35 year old male presents to your office with a painful left forearm mass for the last month. He denies having a known injury or precipitating event. He describes the mass as a superficial nodule that developed around one of his veins. It was sore and swollen over the last few weeks, but symptoms seem to be subsiding over the last few days. On physical exam he appears to have a resolving superficial thrombophlebitis. Figure 1 is a picture of his left forearm. There is no surrounding erythema or warmth or signs of infection and the palpable cord is slightly tender to the touch and firm. What is the next best step in treatment?
Correct
Superficial venous thrombophlebitis (SVT) is caused by inflammation and thrombus of a superficial vein that causes symptoms of a painful erythematous cord like structure. Phlebitis alone is inflammation of the vein without thrombus and is difficult to distinguish from an SVT. SVT is usually found in the lower extremities and associated with varicose veins. In most cases no cause is identified, although trauma such as catheter placement is the most common iatrogenic cause. It is important to educate patients that SVT’s are self-limiting and are not a risk for pulmonary embolisms such as the case of deep vein thrombosis. In rare cases, migratory SVT’s can be a sign of malignancy and higher fevers in elderly patients with an SVT can be a sign of sepsis from an IV line.
The diagnosis of an SVT in the upper extremity is made clinically and imaging is not necessary. SVTs of the lower extremity have a stronger association with DVTs so an ultrasound is often ordered to rule out DVT. Symptoms generally resolve in days to weeks of the initial presentation. Erythema generally subsides first but the palpable nodule of the thrombus can last for months. Treatment involves oral NSAIDs, warm compresses, and time as the symptoms resolve. Thrombectomy is rarely performed and considered when there is progressive thrombus formation toward and deep vein.
Answer A.
References
1. Evans NS, Ratchford EV. Superficial vein thrombosis. Vascular Medicine. 2018 Apr;23(2):187-9.
2. Lee JT, Kalani MA. Treating superficial venous thrombophlebitis. Journal of the National Comprehensive Cancer Network. 2008 Sep 1;6(8):760-5.
Incorrect
Superficial venous thrombophlebitis (SVT) is caused by inflammation and thrombus of a superficial vein that causes symptoms of a painful erythematous cord like structure. Phlebitis alone is inflammation of the vein without thrombus and is difficult to distinguish from an SVT. SVT is usually found in the lower extremities and associated with varicose veins. In most cases no cause is identified, although trauma such as catheter placement is the most common iatrogenic cause. It is important to educate patients that SVT’s are self-limiting and are not a risk for pulmonary embolisms such as the case of deep vein thrombosis. In rare cases, migratory SVT’s can be a sign of malignancy and higher fevers in elderly patients with an SVT can be a sign of sepsis from an IV line.
The diagnosis of an SVT in the upper extremity is made clinically and imaging is not necessary. SVTs of the lower extremity have a stronger association with DVTs so an ultrasound is often ordered to rule out DVT. Symptoms generally resolve in days to weeks of the initial presentation. Erythema generally subsides first but the palpable nodule of the thrombus can last for months. Treatment involves oral NSAIDs, warm compresses, and time as the symptoms resolve. Thrombectomy is rarely performed and considered when there is progressive thrombus formation toward and deep vein.
Answer A.
References
1. Evans NS, Ratchford EV. Superficial vein thrombosis. Vascular Medicine. 2018 Apr;23(2):187-9.
2. Lee JT, Kalani MA. Treating superficial venous thrombophlebitis. Journal of the National Comprehensive Cancer Network. 2008 Sep 1;6(8):760-5.
-
Question 49 of 55
49. Question
A 30 year old male construction worker presents to urgent care with a nail gun injury to the left index finger 2 hours ago. He was holding a piece of wood when the nail missed and hit his finger. A picture of the injury is shown in figure 1. AP and lateral xrays of the finger are shown in figures 2 and 3, respectively. On physical exam the nail appears clean and dry without gross contamination. There is no deformity to the finger. What is the next best step in treatment?
Correct
Nail guns are a common tool used by contractors to quickly drive nails with significant force through building products. The velocity of a nail gun is similar to the force of a .22 caliber handgun or rifle. Nail gun injuries are a common work-related injury, with one study showing nail gun injuries causing 3.9% of workers compensation claims in Ohio and North Carolina. The most common locations of nail gun injuries are the fingers and hand (66%). Nail gun wounds can be complicated by objects on the nail itself such as bards, glues, or oils and by objects dragged in with the nail such as clothing, dirt, and other debris. Xrays should be obtained before nail extraction is attempted to rule out bony involvement and objects that may make simple extraction difficult such as barbs on the nail. In most cases, a nail extraction can be performed with a digital block in the urgent care or emergency room setting without the need for the operating room. A single dose of IV antibiotics, tetanus if indicated, and thorough irrigation of wounds after removal provides a successful treatment option. Extraction with longitudinal force using large needle forceps is generally easy with soft tissue involvement alone but may require larger pliers if the nail is embedded in bone. Before extraction, the portion of the nail at the exit wound should be cleaned before pulling back through the entry wound to remove. A careful exam should be performed upon nail removal to identify any nerve, tendon, or vascular injury. A open exploration in the operating room setting may be necessary with bone involvement or if embedded material is evident that may be retained after extraction.
Answer D.
References
1. Hussey, Keith MB, ChB; Knox, David MB, ChB; Lambah, Alex MB, ChB; Curnier, Alain P. MB, ChB; Holmes, John D. MChir; Davies, Michaela MB, ChB. Nail Gun Injuries to the Hand. The Journal of Trauma: Injury, Infection, and Critical Care 64(1):p 170-173, January 2008. | DOI: 10.1097/TA.0b013e3180d09996
2. Pierpont YN, Pappas-Politis E, Naidu DK, Salas RE, Johnson EL, Payne WG. Nail-gun injuries to the hand. Eplasty. 2008;8:e52. Epub 2008 Nov 13. PMID: 19079574; PMCID: PMC2586285.
Incorrect
Nail guns are a common tool used by contractors to quickly drive nails with significant force through building products. The velocity of a nail gun is similar to the force of a .22 caliber handgun or rifle. Nail gun injuries are a common work-related injury, with one study showing nail gun injuries causing 3.9% of workers compensation claims in Ohio and North Carolina. The most common locations of nail gun injuries are the fingers and hand (66%). Nail gun wounds can be complicated by objects on the nail itself such as bards, glues, or oils and by objects dragged in with the nail such as clothing, dirt, and other debris. Xrays should be obtained before nail extraction is attempted to rule out bony involvement and objects that may make simple extraction difficult such as barbs on the nail. In most cases, a nail extraction can be performed with a digital block in the urgent care or emergency room setting without the need for the operating room. A single dose of IV antibiotics, tetanus if indicated, and thorough irrigation of wounds after removal provides a successful treatment option. Extraction with longitudinal force using large needle forceps is generally easy with soft tissue involvement alone but may require larger pliers if the nail is embedded in bone. Before extraction, the portion of the nail at the exit wound should be cleaned before pulling back through the entry wound to remove. A careful exam should be performed upon nail removal to identify any nerve, tendon, or vascular injury. A open exploration in the operating room setting may be necessary with bone involvement or if embedded material is evident that may be retained after extraction.
Answer D.
References
1. Hussey, Keith MB, ChB; Knox, David MB, ChB; Lambah, Alex MB, ChB; Curnier, Alain P. MB, ChB; Holmes, John D. MChir; Davies, Michaela MB, ChB. Nail Gun Injuries to the Hand. The Journal of Trauma: Injury, Infection, and Critical Care 64(1):p 170-173, January 2008. | DOI: 10.1097/TA.0b013e3180d09996
2. Pierpont YN, Pappas-Politis E, Naidu DK, Salas RE, Johnson EL, Payne WG. Nail-gun injuries to the hand. Eplasty. 2008;8:e52. Epub 2008 Nov 13. PMID: 19079574; PMCID: PMC2586285.
-
Question 50 of 55
50. Question
A 13 year old male presents to your office with severe right ankle pain after an injury 3 days ago. He was being pulled behind a four wheeler on a sled when he fell off and hit his ankle. He had immediate pain and was unable to bear weight. On physical exam he has moderate swelling to the ankle without deformity. He has pain to palpation over the distal tibia and ankle and his pulses and sensation are intact in his foot. AP and lateral xrays are shown in figures 1 and 2. What is the next best step in treatment?
Correct
The patient sustained a distal third tibial diaphyseal fracture with minimal displacement. The fracture pattern appears to end proximal to the physis and does not have intra-articular involvement. A CT would be indicated if an intra-articular injury was suspected. These fractures are very unstable and prone to displacement with any motion or stress. Cast immobilization is the mainstay of treatment for pediatric distal tibia diaphyseal fractures. Prior to cast treatment, closed reduction may be necessary if the fracture is in >10 degrees of varus or valgus alignment or if displaced greater than 3 mm. Treatment of stable, nondisplaced fractures can vary including a boot, long leg cast, or short leg cast. There is no clear consensus on which option is the best and when weight bearing should be initiated. Traditionally patients are placed in a long leg cast with the knee flexed at 30 or 60 degrees to limit or prevent weight bearing. More recent studies have shown short leg casts to be at least as effective as long leg casting in important factors such as time to union and loss of alignment. Barnett et al. found that short leg casting actually offered a decreased time to union compared to long leg casts. Other authors advocate for a non-weight bearing long leg cast initially. Immobilization should extend for at least 6 weeks for distal tibia fractures. Serial xrays should be performed once a week for at least the first three weeks to check for displacement. Early detection of fracture displacement is critical to allow for re-manipulation if necessary. If the fracture starts to heal in a mal-reduced position it is often difficult to correct. 1,2
Answer D.
References
1. Barnett SA, Fontenot B, Leonardi C, Gonzales JA, Gargiulo D, Heffernan MJ. Comparison of short-leg and long-leg casts for the treatment of distal third tibial shaft fractures in children. Journal of Pediatric Orthopaedics. 2021 Mar 1;41(3):e259-65.
2. Raducha JE, Swarup I, Schachne JM, Cruz AI Jr, Fabricant PD. Tibial Shaft Fractures in Children and Adolescents. JBJS Rev. 2019 Feb;7(2):e4.
Incorrect
The patient sustained a distal third tibial diaphyseal fracture with minimal displacement. The fracture pattern appears to end proximal to the physis and does not have intra-articular involvement. A CT would be indicated if an intra-articular injury was suspected. These fractures are very unstable and prone to displacement with any motion or stress. Cast immobilization is the mainstay of treatment for pediatric distal tibia diaphyseal fractures. Prior to cast treatment, closed reduction may be necessary if the fracture is in >10 degrees of varus or valgus alignment or if displaced greater than 3 mm. Treatment of stable, nondisplaced fractures can vary including a boot, long leg cast, or short leg cast. There is no clear consensus on which option is the best and when weight bearing should be initiated. Traditionally patients are placed in a long leg cast with the knee flexed at 30 or 60 degrees to limit or prevent weight bearing. More recent studies have shown short leg casts to be at least as effective as long leg casting in important factors such as time to union and loss of alignment. Barnett et al. found that short leg casting actually offered a decreased time to union compared to long leg casts. Other authors advocate for a non-weight bearing long leg cast initially. Immobilization should extend for at least 6 weeks for distal tibia fractures. Serial xrays should be performed once a week for at least the first three weeks to check for displacement. Early detection of fracture displacement is critical to allow for re-manipulation if necessary. If the fracture starts to heal in a mal-reduced position it is often difficult to correct. 1,2
Answer D.
References
1. Barnett SA, Fontenot B, Leonardi C, Gonzales JA, Gargiulo D, Heffernan MJ. Comparison of short-leg and long-leg casts for the treatment of distal third tibial shaft fractures in children. Journal of Pediatric Orthopaedics. 2021 Mar 1;41(3):e259-65.
2. Raducha JE, Swarup I, Schachne JM, Cruz AI Jr, Fabricant PD. Tibial Shaft Fractures in Children and Adolescents. JBJS Rev. 2019 Feb;7(2):e4.
-
Question 51 of 55
51. Question
A 36 year old female presents to your office with
left wrist pain for 3 months. She has a history of Ehlers–Danlos Syndrome but is otherwise healthy and has not had wrist pain before. She recently took a job in retail and the constant forearm rotation seems to be bothering her left wrist. She occasionally feels a “clunk” with wrist motion and over the last few weeks the wrist feels weaker. On physical exam she has a prominent ulnar dorsally, ulnar sided wrist pain to palpation, and pain with supination and pronation. AP and lateral view xrays of the hand and wrist are shown in figures 1 and 2. What is the most likely cause of this patient’s wrist pain?
Correct
Patients with Ehlers–Danlos Syndrome (EDS) have generalized laxity which can often cause symptoms in the wrist and distal radial ulnar joint (DRUJ) instability. The patient’s xrays show dorsal subluxation of the ulnar and widening of the DRUJ consistent with DRUJ instability. The DRUJ is primary stabilized by soft tissues around the joint including the joint capsule, interosseous membrane, and the triangular fibrocartilage complex (TFCC). DRUJ instability can be due to generalized ligament laxity or a traumatic injury such as a fall on an outstretched hand. Physical exam findings can be subtle with minor ulnar sided pain alone. Severe instability can cause a clunk with rotation of the forearm as the ulnar head dislocates and relocates. Xray findings of DRUJ instability include widening of the radioulnar distance >6mm and dorsal displacement of the ulnar head. Initial treatment focuses on immobilization to reduce forearm rotation and reducing inflammation.
Answer A.
References
1. Omokawa S, Iida A, Kawamura K, Nakanishi Y, Shimizu T, Kira T, Onishi T, Hayami N, Tanaka Y. A biomechanical perspective on distal radioulnar joint instability. Journal of Wrist Surgery. 2017 May;6(02):088-96.
2. Rodríguez-Merchán EC, Shojaie B, Kachooei AR. Distal Radioulnar Joint Instability: Diagnosis and Treatment. Arch Bone Jt Surg. 2022 Jan;10(1):3-16. doi: 10.22038/ABJS.2021.57194.2833. PMID: 35291239; PMCID: PMC8889419.
Incorrect
Patients with Ehlers–Danlos Syndrome (EDS) have generalized laxity which can often cause symptoms in the wrist and distal radial ulnar joint (DRUJ) instability. The patient’s xrays show dorsal subluxation of the ulnar and widening of the DRUJ consistent with DRUJ instability. The DRUJ is primary stabilized by soft tissues around the joint including the joint capsule, interosseous membrane, and the triangular fibrocartilage complex (TFCC). DRUJ instability can be due to generalized ligament laxity or a traumatic injury such as a fall on an outstretched hand. Physical exam findings can be subtle with minor ulnar sided pain alone. Severe instability can cause a clunk with rotation of the forearm as the ulnar head dislocates and relocates. Xray findings of DRUJ instability include widening of the radioulnar distance >6mm and dorsal displacement of the ulnar head. Initial treatment focuses on immobilization to reduce forearm rotation and reducing inflammation.
Answer A.
References
1. Omokawa S, Iida A, Kawamura K, Nakanishi Y, Shimizu T, Kira T, Onishi T, Hayami N, Tanaka Y. A biomechanical perspective on distal radioulnar joint instability. Journal of Wrist Surgery. 2017 May;6(02):088-96.
2. Rodríguez-Merchán EC, Shojaie B, Kachooei AR. Distal Radioulnar Joint Instability: Diagnosis and Treatment. Arch Bone Jt Surg. 2022 Jan;10(1):3-16. doi: 10.22038/ABJS.2021.57194.2833. PMID: 35291239; PMCID: PMC8889419.
-
Question 52 of 55
52. Question
A 70 year old female presents to your clinic with left finger pain and deformity after a fall earlier in the day. On physical exam she has an obvious deviation to the finger and her skin is intact. Figure 1 shows a picture of the finger deformity. Figure 2 is an AP xray of the small finger showing a displaced proximal phalanx fracture. The finger is reduced and figures 3 and 4 show post reduction AP and lateral xrays. The fracture remains minimally displaced with slight extension of the distal fragment of the proximal phalanx. If the proximal fragment further displaces in extension, which complication is most likely to arise?
Correct
Phalanx fractures of the hand are the most common fractures and a frequent reason for an ED or urgent care visit. The surrounding soft tissue around each phalanx can causes deforming forces to these fractures making them very unstable. The flexor and extensor tendons provide an axial force which shortens the proximal phalanx and causes the distal fracture fragment to extend. The interosseous muscles attach to the base of the proximal phalanx which causes flexion of the proximal fracture fragment. These forces can cause fracture displacement after reduction which requires close monitoring with serial weekly xrays for the first 3-4 weeks.
In general non-operative treatment can be accepted if there is less than 20 degrees of angulation on the AP xray, less than 15 degrees in the lateral view, no rotational deformity or shortening, and greater than 50% bony contact. Fracture shortening can cause extensor tendon sagging resulting in an extension lag deformity. Fracture angulation on the lateral view (with the distal fragment in extension) can cause loss of full MCP joint flexion and weak grip strength. Non operative treatment involves immobilization with the MCP in flexion to help reduce the displacing forces of the surrounding soft tissue. The injured finger should be buddy taped to an adjacent finger before placement of a cast or splint to prevent finger rotation. Unstable fractures should be immobilized for up to 4 weeks, at which time the fracture is unlikely to displace and gentle range of motion can begin.
Answer A.
References
1. Lögters TT, Lee HH, Gehrmann S, Windolf J, Kaufmann RA. Proximal Phalanx Fracture Management. Hand (N Y). 2018 Jul;13(4):376-383. doi: 10.1177/1558944717735947. Epub 2017 Oct 27. PMID: 29078727; PMCID: PMC6081790.
2. Balaram AK, Bednar MS. Complications after the fractures of metacarpal and phalanges. Hand clinics. 2010 May 1;26(2):169.
Incorrect
Phalanx fractures of the hand are the most common fractures and a frequent reason for an ED or urgent care visit. The surrounding soft tissue around each phalanx can causes deforming forces to these fractures making them very unstable. The flexor and extensor tendons provide an axial force which shortens the proximal phalanx and causes the distal fracture fragment to extend. The interosseous muscles attach to the base of the proximal phalanx which causes flexion of the proximal fracture fragment. These forces can cause fracture displacement after reduction which requires close monitoring with serial weekly xrays for the first 3-4 weeks.
In general non-operative treatment can be accepted if there is less than 20 degrees of angulation on the AP xray, less than 15 degrees in the lateral view, no rotational deformity or shortening, and greater than 50% bony contact. Fracture shortening can cause extensor tendon sagging resulting in an extension lag deformity. Fracture angulation on the lateral view (with the distal fragment in extension) can cause loss of full MCP joint flexion and weak grip strength. Non operative treatment involves immobilization with the MCP in flexion to help reduce the displacing forces of the surrounding soft tissue. The injured finger should be buddy taped to an adjacent finger before placement of a cast or splint to prevent finger rotation. Unstable fractures should be immobilized for up to 4 weeks, at which time the fracture is unlikely to displace and gentle range of motion can begin.
Answer A.
References
1. Lögters TT, Lee HH, Gehrmann S, Windolf J, Kaufmann RA. Proximal Phalanx Fracture Management. Hand (N Y). 2018 Jul;13(4):376-383. doi: 10.1177/1558944717735947. Epub 2017 Oct 27. PMID: 29078727; PMCID: PMC6081790.
2. Balaram AK, Bednar MS. Complications after the fractures of metacarpal and phalanges. Hand clinics. 2010 May 1;26(2):169.
-
Question 53 of 55
53. Question
A 40 year old male presents to your office with right thumb pain after a fall 2 days ago. He had immediate pain and is unable to grab any objects with the hand now. On physical exam he has mild swelling to the base of the thumb and pain to palpation over the proximal first metacarpal. AP and lateral xrays are shown in figures 1 and 2. This fracture is commonly referred to as which choice below?
Correct
Intra-articular fractures of the base of the thumb are commonly referred to as either Bennett or Rolando fractures. A Rolando fracture breaks off the complete articular surface in a Y or T type pattern from the remaining metacarpal. A Bennett fracture is an intra-articular avulsion fracture of the ulnar aspect of the metacarpal base. Bennett is often referred to as an incomplete articular fracture as only a part of the articular surface is involved. A Bennett fracture is generally unstable as the metacarpal shaft subluxes in a dorsal, proximal, and radial direction due to displacing forces of surrounding tendons and muscles. The anterior oblique ligament holds the ulnar sided fragment in place with its ligament attachment to the trapezium. Complications from this fracture type can arise include early arthritis if the joint isn’t reduced anatomically or hyperextension of the thumb in cases of malunion. Patients can also have loss of thumb motion and decreased grip strength due to poor fracture reduction. For these reasons, intra-articular displacement greater than 2 mm generally requires surgical fixation. Surgical options include closed reduction with percutaneous pinning or open reduction and internal fixation. Non-displaced fractures can be treated with 4 weeks of thumb spica cast immobilization. 1,2
Answer A.
References
1. Carlsen BT, Moran SL. Thumb trauma: Bennett fractures, Rolando fractures, and ulnar collateral ligament injuries. The Journal of hand surgery. 2009 May 1;34(5):945-52.
2. Brownlie C, Anderson D. Bennett Fracture Dislocation: Review and Management. Australian family physician. 2011 Jun;40(6):394-6.
Incorrect
Intra-articular fractures of the base of the thumb are commonly referred to as either Bennett or Rolando fractures. A Rolando fracture breaks off the complete articular surface in a Y or T type pattern from the remaining metacarpal. A Bennett fracture is an intra-articular avulsion fracture of the ulnar aspect of the metacarpal base. Bennett is often referred to as an incomplete articular fracture as only a part of the articular surface is involved. A Bennett fracture is generally unstable as the metacarpal shaft subluxes in a dorsal, proximal, and radial direction due to displacing forces of surrounding tendons and muscles. The anterior oblique ligament holds the ulnar sided fragment in place with its ligament attachment to the trapezium. Complications from this fracture type can arise include early arthritis if the joint isn’t reduced anatomically or hyperextension of the thumb in cases of malunion. Patients can also have loss of thumb motion and decreased grip strength due to poor fracture reduction. For these reasons, intra-articular displacement greater than 2 mm generally requires surgical fixation. Surgical options include closed reduction with percutaneous pinning or open reduction and internal fixation. Non-displaced fractures can be treated with 4 weeks of thumb spica cast immobilization. 1,2
Answer A.
References
1. Carlsen BT, Moran SL. Thumb trauma: Bennett fractures, Rolando fractures, and ulnar collateral ligament injuries. The Journal of hand surgery. 2009 May 1;34(5):945-52.
2. Brownlie C, Anderson D. Bennett Fracture Dislocation: Review and Management. Australian family physician. 2011 Jun;40(6):394-6.
-
Question 54 of 55
54. Question
A 56 year old female presents to your office with left ankle pain for 2 months after an inversion ankle sprain at work. She slipped on ice and had immediate pain to the lateral side of her ankle. Initial AP and lateral xrays are shown in figures 1 and 2. She was diagnosed with an ankle sprain and has been in physical therapy the last 6 weeks without relief. She continues to have pain with walking. MRI done shows an osteochondral lesion to the medial talar done with a subchondral cyst but no surrounding edema (Figure 3). She also has an ununited fracture to the distal aspect of the lateral malleolus (figure 4). What is the best treatment option at this point?
Correct
The patient has a large osteochondral lesion in the medial talus, but the lesion has no surrounding edema indicating the lesion is not an acute injury. The lesion also appears to have stable bone coverage without loose bodies. The only MRI findings consistent with acute pain in this patient is her distal fibula fracture. Given the osteochondral lesion has no surrounding edema this lesion is likely old and not related to the recent ankle sprain. The significance of bone marrow edema in the talus helps determine the acuity of the injury as edema is a source of ankle pain. Bone marrow edema can persist in the talus for 4-6 months, which is a common reason ankle injuries can be painful for that long of duration. Bone edema associated with osteochondral lesions is common in the acute phase and later when the subchondral bone collapses and the condition progresses. Pain associated with osteochondral lesions comes from subchondral bone and its integrity plays an important role in maintaining normal articular cartilage. Treatment decisions are often based on the integrity of subchondral bone and the presence of surrounding edema. Asymptomatic osteochondral lesions can decrease over time, but patients should be advised an ankle injury such as a sprain can aggravate the lesion to become symptomatic. Given the osteochondral lesion appears to be an incidental finding, an ankle stabilizing brace to treat the healing distal fibula fracture is appropriate treatment until symptoms subside. 1,2
Answer A.
References
1. Schachter AK, Chen AL, Reddy PD, Tejwani NC. Osteochondral lesions of the talus. JAAOS-Journal of the American Academy of Orthopaedic Surgeons. 2005 May 1;13(3):152-8.
2. D’Ambrosi R, Maccario C, Ursino C, Serra N, Usuelli FG. The role of bone marrow edema on osteochondral lesions of the talus. Foot and Ankle Surgery. 2018 Jun 1;24(3):229-35.
Incorrect
The patient has a large osteochondral lesion in the medial talus, but the lesion has no surrounding edema indicating the lesion is not an acute injury. The lesion also appears to have stable bone coverage without loose bodies. The only MRI findings consistent with acute pain in this patient is her distal fibula fracture. Given the osteochondral lesion has no surrounding edema this lesion is likely old and not related to the recent ankle sprain. The significance of bone marrow edema in the talus helps determine the acuity of the injury as edema is a source of ankle pain. Bone marrow edema can persist in the talus for 4-6 months, which is a common reason ankle injuries can be painful for that long of duration. Bone edema associated with osteochondral lesions is common in the acute phase and later when the subchondral bone collapses and the condition progresses. Pain associated with osteochondral lesions comes from subchondral bone and its integrity plays an important role in maintaining normal articular cartilage. Treatment decisions are often based on the integrity of subchondral bone and the presence of surrounding edema. Asymptomatic osteochondral lesions can decrease over time, but patients should be advised an ankle injury such as a sprain can aggravate the lesion to become symptomatic. Given the osteochondral lesion appears to be an incidental finding, an ankle stabilizing brace to treat the healing distal fibula fracture is appropriate treatment until symptoms subside. 1,2
Answer A.
References
1. Schachter AK, Chen AL, Reddy PD, Tejwani NC. Osteochondral lesions of the talus. JAAOS-Journal of the American Academy of Orthopaedic Surgeons. 2005 May 1;13(3):152-8.
2. D’Ambrosi R, Maccario C, Ursino C, Serra N, Usuelli FG. The role of bone marrow edema on osteochondral lesions of the talus. Foot and Ankle Surgery. 2018 Jun 1;24(3):229-35.
-
Question 55 of 55
55. Question
An 11 year old obese female presents to your clinic with severe left hip pain after a fall at school 2 hours earlier. Her mother brings her in and mentions she was complaining of vague hip pain over the last few weeks but nothing this severe. She is unable to bear weight on the left hip and is uncomfortable at rest. AP and lateral xrays of the left hip are shown in figures 1 and 2. Recent studies suggest which treatment option would be the best for this patient?
Correct
Slipped Capital Femoral Epiphysis (SCFE) is a common disorder effecting adolescent hips when the growth plate is vulnerable to sheering forces and displacement. Symptoms often start out as vague hip, thigh, or knee pain that gets worse with progressive ambulation. Inability to bear weight on the effected hip is considered an unstable SCFE. Traditional management of unstable SCFE was in situ percutaneous screw fixation. However, in situ fixation does not treat the deformity at the femoral head/neck junction in displaced forms. This has led to the popularization of other procedures such as open and closed reduction with screw fixation to help improve long term outcomes. Another concern of closed management is Increased intracapsular pressure from a hematoma associated with SCFE, which can tamponade the blood supply to the femoral head, increasing the risk of osteonecrosis. For this reason, capsulotomy is often performed after gentle reduction to help evacuate the hematoma and restore perfusion. Timing of reduction, screw fixation, and arthrotomy is controversial. In theory performing a capsulotomy within 24 hours of presentation helps restore perfusion to the femoral head as quickly as possible. Recent studies suggest open and closed reduction of unstable SCFE both have a decreased risk of osteonecrosis over in situ fixation. An optimal reduction also reduces the risk of long-term complications such as femoral acetabular impingement and osteoarthritis. 1,2
Answer C.
References
1. Cazzulino A, Wu W, Allahabadi S, Swarup I. Diagnosis and Management of Unstable Slipped Capital Femoral Epiphysis: A Critical Analysis Review. JBJS Rev. 2021 Jul 16;9(7). doi: 10.2106/JBJS.RVW.20.00268. PMID: 34270502.
2. Wylie JD, Novais EN. Evolving Understanding of and Treatment Approaches to Slipped Capital Femoral Epiphysis. Curr Rev Musculoskelet Med. 2019 Jun;12(2):213-219. doi: 10.1007/s12178-019-09547-5. PMID: 30864075; PMCID: PMC6542912.
Incorrect
Slipped Capital Femoral Epiphysis (SCFE) is a common disorder effecting adolescent hips when the growth plate is vulnerable to sheering forces and displacement. Symptoms often start out as vague hip, thigh, or knee pain that gets worse with progressive ambulation. Inability to bear weight on the effected hip is considered an unstable SCFE. Traditional management of unstable SCFE was in situ percutaneous screw fixation. However, in situ fixation does not treat the deformity at the femoral head/neck junction in displaced forms. This has led to the popularization of other procedures such as open and closed reduction with screw fixation to help improve long term outcomes. Another concern of closed management is Increased intracapsular pressure from a hematoma associated with SCFE, which can tamponade the blood supply to the femoral head, increasing the risk of osteonecrosis. For this reason, capsulotomy is often performed after gentle reduction to help evacuate the hematoma and restore perfusion. Timing of reduction, screw fixation, and arthrotomy is controversial. In theory performing a capsulotomy within 24 hours of presentation helps restore perfusion to the femoral head as quickly as possible. Recent studies suggest open and closed reduction of unstable SCFE both have a decreased risk of osteonecrosis over in situ fixation. An optimal reduction also reduces the risk of long-term complications such as femoral acetabular impingement and osteoarthritis. 1,2
Answer C.
References
1. Cazzulino A, Wu W, Allahabadi S, Swarup I. Diagnosis and Management of Unstable Slipped Capital Femoral Epiphysis: A Critical Analysis Review. JBJS Rev. 2021 Jul 16;9(7). doi: 10.2106/JBJS.RVW.20.00268. PMID: 34270502.
2. Wylie JD, Novais EN. Evolving Understanding of and Treatment Approaches to Slipped Capital Femoral Epiphysis. Curr Rev Musculoskelet Med. 2019 Jun;12(2):213-219. doi: 10.1007/s12178-019-09547-5. PMID: 30864075; PMCID: PMC6542912.